You are on page 1of 109

Chng 1: TNG QUAN V AN TON V BO MT THNG TIN 1.1.

Ni dung ca an ton v bo mt thng tin Khi nhu cu trao i thng tin d liu ngy cng ln v a dng, cc tin b v in t - vin thng v cng ngh thng tin khng ngng c pht trin ng dng nng cao cht lng v lu lng truyn tin th cc quan nim tng v bin php bo v thng tin d liu cng c i mi. Bo v an ton thng tin d liu l mt ch rng, c lin quan n nhiu lnh vc v trong thc t c th c rt nhiu phng php c thc hin bo v an ton thng tin d liu. Cc phng php bo v an ton thng tin d liu c th c quy t vo ba nhm sau: - Bo v an ton thng tin bng cc bin php hnh chnh. - Bo v an ton thng tin bng cc bin php k thut (phn cng). - Bo v an ton thng tin bng cc bin php thut ton (phn mm). Ba nhm trn c th c ng dng ring r hoc phi kt hp. Mi trng kh bo v an ton thng tin nht v cng l mi trng i phng d xn nhp nht l mi trng mng v truyn tin. Bin php hiu qu nht v kinh t nht hin nay trn mng truyn tin v mng my tnh l bin php thut ton. An ton thng tin bao gm cc ni dung sau: - Tnh b mt: tnh kn o ring t ca thng tin - Tnh xc thc ca thng tin, bao gm xc thc i tc( bi ton nhn danh), xc thc thng tin trao i. - Tnh trch nhim: m bo ngi gi thng tin khng th thoi thc trch nhim v thng tin m mnh gi. m bo an ton thng tin d liu trn ng truyn tin v trn mng my tnh c hiu qu th iu trc tin l phi lng trc hoc d on trc cc kh nng khng an ton, kh nng xm phm, cc s c ri ro c th xy ra i vi thng tin d liu c lu tr v trao i trn ng truyn tin cng nh

http://www.ebook.edu.vn

trn mng. Xc nh cng chnh xc cc nguy c ni trn th cng quyt nh c tt cc gii php gim thiu cc thit hi. C hai loi hnh vi xm phm thng tin d liu l: vi phm ch ng v vi phm th ng. Vi phm th ng ch nhm mc ch cui cng l nm bt c thng tin (nh cp thng tin). Vic lm c khi khng bit c ni dung c th nhng c th d ra c ngi gi, ngi nhn nh thng tin iu khin giao thc cha trong phn u cc gi tin. K xm nhp c th kim tra c s lng, di v tn s trao i. V vy vi pham th ng khng lm sai lch hoc hy hoi ni dung thng tin d liu c trao i. Vi phm th ng thng kh pht hin nhng c th c nhng bin php ngn chn hiu qu. Vi phm ch ng l dng vi phm c th lm thay i ni dung, xa b, lm tr, xp xp li th t hoc lm lp li gi tin ti thi im hoc sau mt thi gian. Vi phm ch ng c th thm vo mt s thng tin ngoi lai lm sai lch ni dung thng tin trao i. Vi phm ch ng d pht hin nhng ngn chn hiu qu th kh khn hn nhiu. Mt thc t l khng c mt bin php bo v an ton thng tin d liu no l an ton tuyt i. Mt h thng d c bo v chc chn n u cng khng th m bo l an ton tuyt i.

http://www.ebook.edu.vn

1.2. Cc chin lt an ton h thng : a. Gii hn quyn hn ti thiu (Last Privilege):

y l chin lc c bn nht theo nguyn tc ny bt k mt i tng no cng ch c nhng quyn hn nht nh i vi ti nguyn mng, khi thm nhp vo mng i tng ch c s dng mt s ti nguyn nht nh. b. Bo v theo chiu su (Defence In Depth):

Nguyn tc ny nhc nh chng ta : Khng nn da vo mt ch an ton no d cho chng rt mnh, m nn to nhiu c ch an ton tng h ln nhau. c. Nt tht (Choke Point) :

To ra mt ca khu hp, v ch cho php thng tin i vo h thng ca mnh bng con ng duy nht chnh l ca khu ny. => phi t chc mt c cu kim sot v iu khin thng tin i qua ca ny. d. im ni yu nht (Weakest Link) :

Chin lc ny da trn nguyn tc: Mt dy xch ch chc ti mt duy nht, mt bc tng ch cng ti im yu nht K ph hoi thng tm nhng ch yu nht ca h thng tn cng, do ta cn phi gia c cc yu im ca h thng. Thng thng chng ta ch quan tm n k tn cng trn mng hn l k tip cn h thng, do an ton vt l c coi l yu im nht trong h thng ca chng ta. e. Tnh ton cc:

Cc h thng an ton i hi phi c tnh ton cc ca cc h thng cc b. Nu c mt k no c th b gy mt c ch an ton th chng c th thnh cng bng cch tn cng h thng t do ca ai v sau tn cng h thng t ni b bn trong. f. Tnh a dng bo v :Cn phi s dng nhiu bin php bo v khc nhau cho h thng khc nhau, nu khng c k tn cng vo c mt h thng th chng cng d dng tn cng vo cc h thng khc.
http://www.ebook.edu.vn 3

1.3 Cc mc bo v trn mng : V khng th c mt gii php an ton tuyt i nn ngi ta thng phi s dng ng thi nhiu mc bo v khc nhau to thnh nhiu hng ro chn i vi cc hot ng xm phm. Vic bo v thng tin trn mng ch yu l bo v thng tin ct gi trong my tnh, c bit l cc server trn mng. Bi th ngoi mt s bin php nhm chng tht thot thng tin trn ng truyn mi c gng tp trung vo vic xy dng cc mc ro chn t ngoi vo trong cho cc h thng kt ni vo mng. Thng thng bao gm cc mc bo v sau: a. Quyn truy nhp Lp bo v trong cng l quyn truy nhp nhm kim sot cc ti nguyn ca mng v quyn hn trn ti nguyn . D nhin l kim sot c cc cu trc d liu cng chi tit cng tt. Hin ti vic kim sot thng mc tp. b. ng k tn /mt khu. Thc ra y cng l kim sot quyn truy nhp, nhng khng phi truy nhp mc thng tin m mc h thng. y l phng php bo v ph bin nht v n n gin t ph tn v cng rt hiu qu. Mi ngi s dng mun c tham gia vo mng s dng ti nguyn u phi c ng k tn v mt khu trc. Ngi qun tr mng c trch nhim qun l, kim sot mi hot ng ca mng v xc nh quyn truy nhp ca nhng ngi s dng khc theo thi gian v khng gian (ngha l ngi s dng ch c truy nhp trong mt khong thi gian no ti mt v tr nht nh no ). V l thuyt nu mi ngi u gi kn c mt khu v tn ng k ca mnh th s khng xy ra cc truy nhp tri php. Song iu kh m bo trong thc t v nhiu nguyn nhn rt i thng lm gim hiu qu ca lp bo v ny. C th khc phc bng cch ngi qun mng chu trch nhim t mt khu hoc thay i mt khu theo thi gian. c. M ho d liu bo mt thng tin trn ng truyn ngi ta s dng cc phng php m ho. D liu b bin i t dng nhn thc c sang dng khng nhn thc
http://www.ebook.edu.vn 4

c theo mt thut ton no v s c bin i ngc li trm nhn (gii m). y l lp bo v thng tin rt quan trng. d. Bo v vt l Ngn cn cc truy nhp vt l vo h thng. Thng dng cc bin php truyn thng nh ngn cm tuyt i ngi khng phn s vo phng t my mng, dng kho trn my tnh hoc cc my trm khng c mm. e. Tng la Ngn chn thm nhp tri php v lc b cc gi tin khng mun gi hoc nhn v cc l do no bo v mt my tnh hoc c mng ni b (intranet)

Tng la (Fire Walls) Bo vt l (Physical protect)

M ho d liu (Data Encryption)

ng k v mt khu (Login/Password)

Quyn truy nhp (Access Rights) hnh a: ccThng tin (Information) my tnh ,c bo v trn mng

f. Qun tr mng. Trong thi i pht trin ca cng ngh thng tin, mng my tnh quyt nh ton b hot ng ca mt c quan, hay mt cng ty x nghip. V vy vic bo m cho h thng mng my tnh hot ng mt cch an ton, khng xy ra s c l mt cng vic cp thit hng u. Cng tc qun tr mng my tnh phi c thc hin mt cch khoa hc m bo cc yu cu sau :
http://www.ebook.edu.vn 5

Mc bo v

- Ton b h thng hot ng bnh thng trong gi lm vic. - C h thng d phng khi c s c v phn cng hoc phn mm xy ra. - Backup d liu quan trng theo nh k. - Bo dng mng theo nh k. - Bo mt d liu, phn quyn truy cp, t chc nhm lm vic trn mng. 1.4. An ton thng tin bng mt m Mt m l mt ngnh khoa hc chuyn nghin cu cc phng php truyn tin b mt. Mt m bao gm : Lp m v ph m. Lp m bao gm hai qu trnh: m ha v gii m. bo v thng tin trn ng truyn ngi ta thng bin i n t dng nhn thc c sang dng khng nhn thc c trc khi truyn i trn mng, qu trnh ny c gi l m ho thng tin (encryption), trm nhn phi thc hin qu trnh ngc li, tc l bin i thng tin t dng khng nhn thc c (d liu c m ho) v dng nhn thc c (dng gc), qu trnh ny c gi l gii m. y l mt lp bo v thng tin rt quan trng v c s dng rng ri trong mi trng mng. bo v thng tin bng mt m ngi ta thng tip cn theo hai hng: - Theo ng truyn (Link_Oriented_Security). - T nt n nt (End_to_End). Theo cch th nht thng tin c m ho bo v trn ng truyn gia hai nt m khng quan tm n ngun v ch ca thng tin . y ta lu rng thng tin ch c bo v trn ng truyn, tc l mi nt u c qu trnh gii m sau m ho truyn i tip, do cc nt cn phi c bo v tt. Ngc li theo cch th hai thng tin trn mng c bo v trn ton ng truyn t ngun n ch. Thng tin s c m ho ngay sau khi mi to ra v ch c gii m khi v n ch. Cch ny mc phi nhc im l
http://www.ebook.edu.vn 6

ch c d liu ca ngi

ung th mi c th m ha c cn d liu iu

khin th gi nguyn c th x l ti cc nt. 1.5. Vai tr ca h mt m Cc h mt m phi thc hin c cc vai tr sau: - H mt m phi che du c ni dung ca vn bn r (PlainText) m bo sao cho ch ngi ch hp php ca thng tin mi c quyn truy cp thng tin (Secrety), hay ni cch khc l chng truy nhp khng ng quyn hn. - To cc yu t xc thc thng tin, m bo thng tin lu hnh trong h thng n ngi nhn hp php l xc thc (Authenticity). - T chc cc s ch k in t, m bo khng c hin tng gi mo, mo danh gi thng tin trn mng. u im ln nht ca bt k h mt m no l c th nh gi c phc tp tnh ton m k ch phi gii quyt bi ton c th ly c thng tin ca d liu c m ho. Tuy nhin mi h mt m c mt s u v nhc im khc nhau, nhng nh nh gi c phc tp tnh ton m ta c th p dng cc thut ton m ho khc nhau cho tng ng dng c th tu theo d yu cu v an ton. Cc thnh phn ca mt h mt m : nh ngha : Mt h mt l mt b 5 (P,C,K,E,D) tho mn cc iu kin sau: - P l mt tp hp hu hn cc bn r (PlainText), n c gi l khng gian bn r. - C l tp cc hu hn cc bn m (Crypto), n cn c gi l khng gian cc bn m. Mi phn t ca C c th nhn c bng cch p dng php m ho Ek ln mt phn t ca P, vi k K. - K l tp hu hn cc kho hay cn gi l khng gian kho. i vi mi phn t k ca K c gi l mt kho (Key). S lng ca khng gian kho
http://www.ebook.edu.vn 7

phi ln k ch: khng c thi gian th mi kho c th (phng php vt cn). - i vi mi k K c mt quy tc m eK: P C v mt quy tc gii m tng ng dK D. Mi eK: P C v dK: C P l nhng hm m: dK (eK(x))=x vi mi bn r x P.

Bn r

M ho

Bn m

Gii m

Bn r

Kho

M ho vi kho m v kho gii ging nhau

1.6. Phn loi h mt m C nhiu cch phn loi h mt m. Da vo cch truyn kha c th phn cc h mt m thnh hai loi: - H mt i xng (hay cn gi l mt m kha b mt): l nhng h mt dung chung mt kho c trong qu trnh m ho d liu v gii m d liu. Do kho phi c gi b mt tuyt i. - H mt m bt i xng (hay cn gi l mt m kha cng khai) : Hay cn gi l h mt m cng khai, cc h mt ny dng mt kho m ho sau dng mt kho khc gii m, ngha l kho m ho v gii m l khc nhau. Cc kho ny to nn tng cp chuyn i ngc nhau v khng c kho no c th suy c t kho kia. Kho dng m ho c th cng khai nhng kho dng gii m phi gi b mt.

http://www.ebook.edu.vn

Ngoi ra nu da vo thi gian a ra h mt m ta cn c th phn lm hai loi: Mt m c in (l h mt m ra i trc nm 1970) v mt m hin i (ra i sau nm 1970). Cn nu da vo cch thc tin hnh m th h mt m cn c chia lm hai loi l m dng (tin hnh m tng khi d liu, mi khi li da vo cc kha khc nhau, cc kha ny c sinh ra t hm sinh kha, c gi l dng kha ) v m khi (tin hnh m tng khi d liu vi kha nh nhau) 1.7. Tiu chun nh gi h mt m nh gi mt h mt m ngi ta thng nh gi thng qua cc tnh cht sau: a, an ton: Mt h mt c a vo s dng iu u tin phi c an ton cao. u im ca mt m l c th nh gi c an ton thng qua an ton tnh ton m khng cn phi ci t. Mt h mt c coi l an ton nu ph h mt m ny phi dng n php ton. M gii quyt n php ton cn thi gian v cng ln, khng th chp nhn c. Mt h mt m c gi l tt th n cn phi m bo cc tiu chun sau: - Chng phi c phng php bo v m ch da trn s b mt ca cc kho, cng khai thut ton. - Khi cho kho cng khai eK v bn r P th chng ta d dng tnh c eK(P) = C. Ngc li khi cho dK v bn m C th d dng tnh c dK(M)=P. Khi khng bit dK th khng c kh nng tm c M t C, ngha l khi cho hm f: X Y th vic tnh y=f(x) vi mi x X l d cn vic tm x khi bit y li l vn kh v n c gi l hm mt chiu. - Bn m C khng c c cc c im gy ch , nghi ng. b, Tc m v gii m: Khi nh gi h mt m chng ta phi ch n tc m v gii m. H mt tt th thi gian m v gii m nhanh.

http://www.ebook.edu.vn

c, Phn phi kha: Mt h mt m ph thuc vo kha, kha ny c truyn cng khai hay truyn kha b mt. Phn phi kha b mt th chi ph s cao hn so vi cc h mt c kha cng khai. V vy y cng l mt tiu ch khi la chn h mt m.

http://www.ebook.edu.vn

10

Chng 2: CC PHNG PHP M HA C IN 2.1. Cc h mt m c in 2.1.1. M dch vng ( shift cipher) Phn ny s m t m dch (MD) da trn s hc theo modulo. Trc tin s im qua mt s nh ngha c bn ca s hc ny. nh ngha Gi s a v b l cc s nguyn v m l mt s nguyn dng. Khi ta vit a b (mod m) nu m chia ht cho b-a. Mnh a b (mod m) c gi l " a ng d vi b theo modulo m". S nguyn m c gi l mudulus. Gi s chia a v b cho m v ta thu c phn thng nguyn v phn d, cc phn d nm gia 0 v m-1, ngha l a = q1m + r1 v b = q2m + r2 trong 0 r1 m-1 v 0 r2 m-1. Khi c th d dng thy rng a b (mod m) khi v ch khi r1 = r2 . Ta s dng k hiu a mod m (khng dng cc du ngoc) xc nh phn d khi a c chia cho m (chnh l gi tr r1 trn). Nh vy: a b (mod m) khi v ch khi a mod m = b mod m. Nu thay a bng a mod m th ta ni rng a c rt gn theo modulo m. Nhn xt: Nhiu ngn ng lp trnh ca my tnh xc nh a mod m l phn d trong di - m+1,.. ., m-1 c cng du vi a. V d -18 mod 7 s l -4, gi tr ny khc vi gi tr 3 l gi tr c xc nh theo cng thc trn. Tuy nhin, thun tin ta s xc nh a mod m lun l mt s khng m. By gi ta c th nh ngha s hc modulo m: Zm c coi l tp hp {0,1,. . .,m-1} c trang b hai php ton cng v nhn. Vic cng v nhn trong Zm c thc hin ging nh cng v nhn cc s thc ngoi tr mt im l cc kt qu c rt gn theo modulo m. V d tnh 11 13 trong Z16 . Tng t nh vi cc s nguyn ta c 11 13 = 143. rt gn 143 theo modulo 16, ta thc hin php chia bnh thng: 143 = 8 16 + 15, bi vy 143 mod 16 = 15 trong Z16 .

http://www.ebook.edu.vn

11

Cc nh ngha trn php cng v php nhn Zm tho mn hu ht cc quy tc quen thuc trong s hc. Sau y ta s lit k m khng chng minh cc tnh cht ny: 1. 2. Php cng l ng, tc vi bt k a,b Zm ,a +b Zm Php cng l giao hon, tc l vi a,b bt k Zm

a+b = b+a 3. Php cng l kt hp, tc l vi bt k a,b,c Zm

(a+b)+c = a+(b+c) 4. 0 l phn t n v ca php cng, c ngha l vi a bt k Zm

a+0 = 0+a = a 5. Phn t nghch o ca php cng ca phn t bt k (a Zm ) l m-a,

ngha l a+(m-a) = (m-a)+a = 0 vi bt k a Zm . 6. 7. 8. 9. Php nhn l ng , tc l vi a,b bt k Zm , ab Zm . Php nhn l giao hon , ngha l vi a,b bt k Zm , ab = ba Php nhn l kt hp, ngha l vi a,b,c Zm , (ab)c = a(cb) 1 l phn t n v ca php nhn, tc l vi bt k a Zm

a1 = 1a = a 10. Php nhn c tnh cht phn phi i vi php cng, tc l i vi a,b,c Zm , (a+b)c = (ac)+(bc) v a(b+c) = (ab) + (ac) Cc tnh cht 1,3-5 ni ln rng Zm lp nn mt cu trc i s c gi l mt nhm theo php cng. V c thm tnh cht 4 nhm c gi l nhm Aben (hay nhm giao hon). Cc tnh cht 1-10 s thit lp nn mt vnh Zm . Mt s v d quen thuc ca vnh l cc s nguyn Z, cc s thc R v cc s phc C. Tuy nhin cc vnh ny u v hn, cn mi quan tm ca chng ta ch gii hn trn cc vnh hu hn.
http://www.ebook.edu.vn 12

V phn t ngc ca php cng tn ti trong Zm nn cng c th tr cc phn t trong Zm . Ta nh ngha a-b trong Zm l a+m-b mod m. Mt cch tng t c th tnh s nguyn a-b ri rt gon theo modulo m. V d : tnh 11-18 trong Z31, ta tnh 11+31 18 mod 31= 11+13 mod 31 = 24. Ngc li, c th ly 11-18 c -7 ri sau tnh -7 mod 31 =31-7= 24. M dch vng c xc nh trn Z26 (do c 26 ch ci trn bng ch ci ting Anh) mc d c th xc nh n trn Zm vi modulus m tu . D dng thy rng, MDV s to nn mt h mt nh xc nh trn, tc l dK(eK(x)) = x vi mi x Z26 . Ta c s m nh sau:
Gi s P = C = K = Z26 vi 0 k 25 , nh ngha: eK(x) = x +K mod 26 v dK(x) = y -K mod 26 (x,y Z26)

Nhn xt: Trong trng hp K = 3, h mt thng c gi l m Caesar tng c Julius Caesar s dng. Ta s s dng MDV (vi modulo 26) m ho mt vn bn ting Anh thng thng bng cch thit lp s tng ng gia cc k t v cc thng d theo modulo 26 nh sau: A 0,B 1, . . ., Z 25. V php tng ng ny cn dng trong mt vi v d nn ta s ghi li cn tin dng sau ny: Sau y l mt v d nh minh ho V d 1.1: Gi s kho cho MDV l K = 11 v bn r l: wewillmeetatmidnight Trc tin bin i bn r thnh dy cc s nguyn nh dng php tng ng trn. Ta c:

http://www.ebook.edu.vn

13

22 0

4 19

22 12

8 8

11 3

11 13

12 8

4 6

4 7

19 19

sau cng 11 vo mi gi tr ri rt gn tng theo modulo 26 7 11 15 4 7 23 19 19 22 14 22 24 23 19 15 17 15 18 4 4

Cui cng bin i dy s nguyn ny thnh cc k t thu c bn m sau: HPHTWWXPPELEXTOYTRSE gi m bn m ny, trc tin, Bob s bin i bn m thnh dy cc s nguyn ri tr i gi tr cho 11 ( rt gn theo modulo 26) v cui cng bin i li dy ny thnh cc k t. Nhn xt: Trong v d trn, ta dng cc ch in hoa cho bn m, cc ch thng cho bn r tin phn bit. Quy tc ny cn tip tc s dng sau ny. Nu mt h mt c th s dng c trong thc t th n pho tho mn mt s tnh cht nht nh. Ngay sau y s nu ra hai trong s : 1. Mi hm m ho eK v mi hm gii m dK phi c kh nng tnh ton c mt cch hiu qu. 2. i phng da trn xu bn m phi khng c kh nng xc nh kho K dng hoc khng c kh nng xc nh c xu bn r x. Tnh cht th hai xc nh (theo cch kh mp m) tng "bo mt". Qu trnh th tnh kho K (khi bit bn m y) c gi l m thm (sau ny khi nim ny s c lm chnh xc hn). Cn ch rng, nu Oscar c th xc nh c K th anh ta c th gii m c y nh Bob bng cch dng dK. Bi vy, vic xc nh K ch t cng kh nh vic xc nh bn r x. Nhn xt rng, MDV (theo modulo 26) l khng an ton v n c th b thm theo phng php vt cn. Do ch c 26 kho nn d dng th mi kho dK

http://www.ebook.edu.vn

14

c th cho ti khi nhn c bn r c ngha. iu ny c minh ho theo v d sau: V du 1.2 Cho bn m JBCRCLQRWCRVNBJENBWRWN ta s th lin tip cc kho gii m d0 ,d1 .. . v y thu c:

http://www.ebook.edu.vn

15

jbcrclqrwcrvnbjenbwrwn iabqbkpqvbqumaidmavqvm hzapajopuaptlzhclzupul gyzozinotzoskygbkytotk jxynyhmnsynrjexfajxsnsj ewxmxglmrxmqiweziwrmri dvwlwfklqwlphvodyhvqlqh cuvkvejkpvkogucxgupkpg btujudijoujnftbwfojof astitchintimesavesnine Ti y ta xc nh c bn r v dng li. Kho tng ng K = 9. Trung bnh c th tnh c bn r sau khi th 26/2 = 13 quy tc gii m. Nh ch ra trong v d trn, iu kin mt h mt an ton l php tm kho vt cn phi khng th thc hin c, tc khng gian kho phi rt ln. Tuy nhin, mt khng gian kho ln vn cha m bo mt. 2.1.2. M thay th Mt h mt ni ting khc l h m thay th. H mt ny c s dng hng trm nm. Tr chi ch "cryptogram" trong cc bi bo l nhng v d v MTT. Trn thc t MTT c th ly c P v C u l b ch ci ting anh, gm 26 ch ci. Ta dng Z26 trong MDV v cc php m v gii m u l cc php ton i s. Tuy nhin, trong MTT, thch hp hn l xem php m v gii m nh cc hon v ca cc k t. M thay th
Cho P =C = Z26 . K cha mi hon v c th ca 26 k hiu 0,1, . . . ,25 Vi mi php hon v K , ta nh ngha: e(x) = (x) v d(y) = -1(y) trong -1 l hon v ngc ca . http://www.ebook.edu.vn 16

Sau y l mt v d v php hon v ngu nhin to nn mt hm m ho (cng nh trc, cc k hiu ca bn r c vit bng ch thng cn cc k hiu ca bn m l ch in hoa). Nh vy, e (a) = X, e (b) = N,. . . . Hm gii m l php hon v ngc. iu ny c thc hin bng cch vit hng th hai ln trc ri sp xp theo th t ch ci. Ta nhn c: Bi vy d (A) = d, d(B) = 1, . . . V d: Hy gii m bn m: M G Z V Y Z L G H C M H J M Y X S S E M N H A H Y C D L M H A. Mi kho ca MTT l mt php hon v ca 26 k t. S cc hon v ny l 26!, ln hn 4 10 26 l mt s rt ln. Bi vy, php tm kho vt cn khng th thc hin c, thm ch bng my tnh. Tuy nhin, sau ny s thy rng MTT c th d dng b thm bng cc phng php khc. 2.1.3. M Affine MDV l mt trng hp c bit ca MTT ch gm 26 trong s 26! Cc hon v c th ca 26 phn t. Mt trng hp c bit khc ca MTT l m Affine c m t di y. Trong m Affine, ta gii hn ch xt cc hm m c dng: e(x) = ax + b mod 26 a, b Z26 . Cc hm ny c gi l cc hm Affine (ch rng khi a = 1, ta c MDV). vic gii m c th thc hin c, yu cu cn thit l hm Affine phi l n nh. Ni cch khc, vi bt k y Z26, ta mun c ng nht thc sau: ax + b y (mod 26) phi c nghim x duy nht. ng d thc ny tng ng vi: ax y-b (mod 26)
http://www.ebook.edu.vn 17

V y thay i trn Z26 nn y-b cng thay i trn Z26 . Bi vy, ta ch cn nghin cu phng trnh ng d: ax y (mod 26) (y Z26 ).

Ta bit rng, phng trnh ny c mt nghim duy nht i vi mi y khi v ch khi UCLN(a,26) = 1 ( y hm UCLN l c chung ln nht ca cc bin ca n). Trc tin ta gi s rng, UCLN(a,26) = d >1. Khi , ng d thc ax 0 (mod 26) s c t nht hai nghim phn bit trong Z26 l x = 0 v x = 26/d. Trong trng hp ny, e(x) = ax + b mod 26 khng phi l mt hm n nh v bi vy n khng th l hm m ho hp l. V d, do UCLN(4,26) = 2 nn 4x +7 khng l hm m ho hp l: x v x+13 s m ho thnh cng mt gi tr i vi bt k x Z26 . Ta gi thit UCLN(a,26) = 1. Gi s vi x1 v x2 no tho mn: ax1 ax2 (mod 26) Khi a(x1- x2) 0(mod 26) bi vy 26 | a(x1- x2) By gi ta s s dng mt tnh cht ca php chia sau: Nu UCLN(a,b)=1 v a bc th a c. V 26 a(x1- x2) v UCLN(a,26) = 1 nn ta c: 26(x1- x2) tc l x1 x2 (mod 26) Ti y ta chng t rng, nu UCLN(a,26) = 1 th mt ng d thc dng ax y (mod 26) ch c (nhiu nht) mt nghim trong Z26 . Do , nu ta cho x thay i trn Z26 th ax mod 26 s nhn c 26 gi tr khc nhau theo modulo 26 v ng d thc ax y (mod 26) ch c mt nghim y duy nht.

http://www.ebook.edu.vn

18

Khng c g c bit i v s 26 trong khng nh ny. Bi vy, bng cch tng t ta c th chng minh c kt qu sau: nh l ng d thc ax b mod m ch c mt nghim duy nht x Zm vi mi b

Zm khi v ch khi UCLN(a,m) = 1.


V 26 = 2 13 nn cc gi tr a Z26 tho mn UCLN(a,26) = 1 l a = 1, 3, 5, 7, 9, 11, 13, 15, 17, 19, 21, 23 v 25. Tham s b c th l mt phn t bt k trong Z26 . Nh vy, m Affine c 12 26 = 312 kho c th (d nhin con s ny qu nh bo m an ton). By gi ta s xt bi ton chung vi modulo m. Ta cn mt nh ngha khc trong l thuyt s. nh ngha Gi s a 1 v m 2 l cc s nguyn. UCLN(a,m) = 1 th ta ni rng a v m l nguyn t cng nhau. S cc s nguyn trong Zm nguyn t cng nhau vi m thng c k hiu l (m) (hm ny c gi l hm Euler). Mt kt qu quan trng trong l thuyt s cho ta gi tr ca (m) theo cc tha s trong php phn tch theo lu tha cc s nguyn t ca m. (Mt s nguyn p >1 l s nguyn t nu n khng c c dng no khc ngoi 1 v p. Mi s nguyn m >1 c th phn tch c thnh tch ca cc lu tha cc s nguyn t theo cch duy nht. V d 60 = 2 3 3 5 v 98 = 2 7 2 ). S kho trong m Affine trn Zm bng (m), trong (m) c cho theo cng thc trn. (S cc php chn ca b l m v s cc php chn ca a l (m) vi hm m ho l e(x) = ax + b). V d, khi m = 60, (60)=(5.22.3)=(5). (22). (3) = 2 2 4 = 16 v s cc kho trong m Affine l 960. (xem tnh cht ca hm phi euler chng 4) By gi ta s xt xem cc php ton gii m trong mt m Affine vi modulo m = 26. Gi s UCLN(a,26) = 1. gii m cn gii phng trnh ng d y ax+b (mod 26) theo x. T tho lun trn thy rng, phng trnh ny c
http://www.ebook.edu.vn 19

mt nghim duy nht trong Z26 . Tuy nhin ta vn cha bit mt phng php hu hiu tm nghim. iu cn thit y l c mt thut ton hu hiu lm vic . Rt may l mt s kt qu tip sau v s hc modulo s cung cp mt thut ton gii m hu hiu cn tm. nh ngha: Gi s a Zm . Phn t nghch o (theo php nhn) ca a l phn t a-1 Zm sao cho aa-1 a-1 1 (mod m). Bng cc l lun tng t nh trn, c th chng t rng a c nghch o theo modulo m khi v ch khi UCLN(a,m) =1, v nu nghch o ny tn ti th n phi l duy nht. Ta cng thy rng, nu b = a-1 th a = b-1 . Nu p l s nguyn t th mi phn t khc khng ca ZP u c nghch o. Mt vnh trong mi phn t u c nghch o c gi l mt trng. Trong phn sau s m t mt thut ton hu hiu tnh cc nghch o ca Zm vi m tu . Tuy nhin, trong Z26, ch bng phng php th v sai cng c th tm c cc nghch o ca cc phn t nguyn t cng nhau vi 26: 1-1 = 1, 3-1 = 9, 5-1 = 21, 7-1 = 15, 11-1 = 19, 17-1 =23, 25-1 = 25. (C th d dng kim chng li iu ny, v d: 7 15 = 105 1 mod 26, bi vy 7-1 = 15). Xt phng trnh ng d y ax+b (mod 26). Phng trnh ny tng ng vi ax y-b ( mod 26) V UCLN(a,26) =1 nn a c nghch o theo modulo 26. Nhn c hai v ca ng d thc vi a-1 ta c: a-1(ax) a-1(y-b) (mod 26) p dng tnh kt hp ca php nhn modulo: a-1(ax) (a-1a)x 1x x. Kt qu l x a-1(y-b) (mod 26). y l mt cng thc tng minh cho x. Nh vy hm gii m l:

http://www.ebook.edu.vn

20

d(y) = a-1(y-b) mod 26 Cho m t y v m Affine. Sau y l mt v d nh


Cho P = C = Z26 v gi s P = { (a,b) Z26 Z26 : UCLN(a,26) =1 } Vi K = (a,b) K , ta nh ngha: eK(x) = ax +b mod 26 v dK(y) = a-1(y-b) mod 26, x,y Z26

Mt mA ffine V d: Gi s K = (7,3). Nh nu trn, 7-1 mod 26 = 15. Hm m ho l eK(x) = 7x+3 V hm gii m tng ng l: dK(x) = 15(y-3) = 15y -19 y, tt c cc php ton u thc hin trn Z26. Ta s kim tra liu dK(eK(x)) = x vi mi x Z26 khng? Dng cc tnh ton trn Z26 , ta c dK(eK(x)) =dK(7x+3) =15(7x+3)-19 = x +45 -19= x. minh ho, ta hy m ho bn r hot. Trc tin bin i cc ch h, o, t thnh cc thng du theo modulo 26. Ta c cc s tng ng l 7, 14 v 19. By gi s m ho: 7 7 +3 mod 26 = 52 mod 26 = 0 7 14 + 3 mod 26 = 101 mod 26 =23 7 19 +3 mod 26 = 136 mod 26 = 6 Bi vy 3 k hiu ca bn m l 0, 23 v 6 tng ng vi xu k t AXG. Vic gii m s do bn c thc hin nh mt bi tp.

http://www.ebook.edu.vn

21

2.1.4. M Vigenre Trong c hai h MDV v MTT (mt khi kho c chn) mi k t s c nh x vo mt k t duy nht. V l do , cc h mt cn c gi h thay th n biu. By gi ta s trnh by mt h mt khng phi l b ch n, l h m Vigenre ni ting. Mt m ny ly tn ca Blaise de Vigenre sng vo th k XVI. S dng php tng ng A 0, B 1, . . . , Z 25 m t trn, ta c th gn cho mi khoa K vi mt chui k t c di m c gi l t kho. Mt m Vigenre s m ho ng thi m k t: Mi phn t ca bn r tng ng vi m k t. Xt mt v d: Gi s m =6 v t kho l CIPHER. T kho ny tng ng vi dy s K = (2,8,15,4,17). Gi s bn r l xu: Thiscryptosystemisnotsecure
Cho m l mt s nguyn dng c nh no . nh ngha P = C = K = (Z26)m . Vi kho K = (k1, k2, . . . ,km) ta xc nh : eK(x1, x2, . . . ,xm) = (x1+k1, x2+k2, . . . , xm+km) v dK(y1, y2, . . . ,ym) = (y1-k1, y2-k2, . . . , ym-km) trong tt c cc php ton c thc hin trong Z26

Mt m Vigenre Ta s bin i cc phn t ca bn r thnh cc thng d theo modulo 26, vit chng thnh cc nhm 6 ri cng vi t kho theo modulo 26 nh sau:
19 2 21 18 2 20 7 8 15 19 8 1 8 15 23 4 15 19 18 7 25 12 7 19 2 4 6 8 4 12 17 17 8 18 17 9 24 2 0 13 2 15 15 8 23 14 8 22 19 15 8 19 15 8 14 7 21 18 7 15 18 4 22 4 4 8 24 17 15 2 17 19

20 17 4 2 8 15 http://www.ebook.edu.vn 22 25 19

22

Bi vy, dy k t tng ng ca xu bn m s l:V P X Z G I A X I V W PUBTTMJPWIZITWZT gii m ta c th dng cng t kho nhng thay cho cng, ta tr cho n theo modulo 26. Ta thy rng cc t kho c th vi s di m trong mt m Vigenre l 26m, bi vy, thm ch vi cc gi tr m kh nh, phng php tm kim vt cn cng yu cu thi gian kh ln. V d, nu m = 5 th khng gian kho cng c kch thc ln hn 1,1 107 . Lng kho ny ln ngn nga vic tm kho bng tay (ch khng phi dng my tnh). Trong h mt Vigenre c t kho di m, mi k t c th c nh x vo trong m k t c th c (gi s rng t kho cha m k t phn bit). Mt h mt nh vy c gi l h mt thay th a biu (polyalphabetic). Ni chung, vic thm m h thay th a biu s kh khn hn so vic thm m h n biu. 2.1.5. Mt m Hill Trong phn ny s m t mt h mt thay th a biu khc c gi l mt m Hill. Mt m ny do Lester S.Hill a ra nm 1929. Gi s m l mt s nguyn dng, t P = C = (Z26)m . tng y l ly m t hp tuyn tnh ca m k t trong mt phn t ca bn r to ra m k t mt phn t ca bn m. V d nu m = 2 ta c th vit mt phn t ca bn r l x = (x1,x2) v mt phn t ca bn m l y = (y1,y2), y, y1cng nh y2 u l mt t hp tuyn tnh ca x1 v x2. Chng hn, c th ly y1 = 11x1+ 3x2 y2 = 8x1+ 7x2 Tt nhin c th vit gn hn theo k hiu ma trn nh sau
http://www.ebook.edu.vn 23

11 8 (y1 y2) = (x1 x2) 3 7

Ni chung, c th ly mt ma trn K kch thc m m lm kho. Nu mt phn t hng i v ct j ca K l ki,j th c th vit K = (ki,j), vi x = (x1, x2, . . . ,xm) P v K K , ta tnh y = eK(x) = (y1, y2, . . . ,ym) nh sau:

(y1,. . .,ym) (x1, . . . ,xm)

k1,1 k1,2 ... k2,1 k2,2 ... ... ... ... km,1 km,2 ...

k1,m k2,m .. km,m

Ni mt cch khc y = xK. Chng ta ni rng bn m nhn c t bn r nh php bin i tuyn tnh. Ta s xt xem phi thc hin gii m nh th no, tc l lm th no tnh x t y. Bn c lm quen vi i s tuyn tnh s thy rng phi dng ma trn nghch o K-1 gi m. Bn m c gii m bng cng thc y K-1 . Sau y l mt s nh ngha v nhng khi nim cn thit ly t i s tuyn tnh. Nu A = (xi,j) l mt ma trn cp l m v B = (b1,k ) l mt ma trn
m

c1,k =

i,j bj,k

j=1

cp m n th tch ma trn AB = (c1,k ) c nh ngha theo cng thc: Vi 1 i l v 1 k l. Tc l cc phn t hng i v ct th k ca AB c to ra bng cch ly hng th i ca A v ct th k ca B, sau nhn tng ng cc phn t vi nhau v cng li. Cn rng AB l mt ma trn cp l n.

http://www.ebook.edu.vn

24

Theo nh ngha ny, php nhn ma trn l kt hp (tc (AB)C = A(BC)) nhng khng giao hon (khng phi lc no AB = BA, thm ch i vi ma trn vung A v B). Ma trn n v m m (k hiu l Im ) l ma trn cp m m c cc s 1 nm ng cho chnh v cc s 0 v tr cn li. Ma trn n v cp 2 l:
1 0 0 1

Im c gi l ma trn n v v AIm = A vi mi ma trn cp l m v ImB =B vi mi ma trn cp m n. Ma trn nghch o ca ma trn A cp m m (nu tn ti) l ma trn A-1 sao cho AA-1 = A-1A = Im . Khng phi mi ma trn u c nghch o, nhng nu tn ti th n duy nht. Vi cc nh ngha trn, c th d dng xy dng cng thc gii m nu: V y = xK, ta c th nhn c hai v ca ng thc vi K-1 v nhn c: yK-1 = (xK)K-1 = x(KK-1) = xIm = x ( Ch s dng tnh cht kt hp)

12 8 3 7

-1 =

8 18 23 11

C th thy rng, ma trn m ho trn c nghch o trong Z26: V

(Hy nh rng18 php117+823 1118+811 thc hin t ton s hc u c 11 8 7 mi


3 7 23 11 = 37+723 318+711

261 286 182 131

1 0 0 1

(theo modulo 26). Sau y l mt v d minh ho cho vic m ho v gii m trong h mt m Hill.
http://www.ebook.edu.vn 25

V d:
= 11 8 3 7

Gi s kho

K-1 =

7 18 23 11

(9,20)

11 8 3 7

= (99+60, 72+140) = (3,4)

T cc tnh ton trn ta c: Gi s cn m ho bn r "July". Ta c hai phn t ca bn r m ho: (9,20) (ng vi Ju) v (11,24) (ng vi ly). Ta tnh nh sau: Bi vy bn m ca July l DELW. gii m Bob s tnh

Nh vy Bob nhn c bn ng.


11 8 7 18 3 7 23 11

(11,24) (3,4)

= (121+72, 88+168) = (11,22) = (9,20)

Cho ti lc ny ta ch ra rng c th thc hin php gii m nu K c


7 18 23 11

(11,22)

= (11,24)

mt nghch o. Trn thc t, php gii m l c th thc hin c, iu kin cn l K phi c nghch o. (iu ny d dng rt ra t i s tuyn tnh
http://www.ebook.edu.vn 26

s cp, tuy nhin s khng chng minh y). Bi vy, chng ta ch quan tm ti cc ma trn K kh nghich. Tnh kh nghch ca mt ma trn vung ph thuc vo gi tr nh thc ca n. trnh s tng qut ho khng cn thit, ta ch gii hn trong trng hp 22. nh ngha nh thc ca ma trn A = (a,i j ) cp 2 2 l gi tr det A = a1,1 a2,2 - a1,2 a2,1 Nhn xt: nh thc ca mt ma trn vung cp mxm c th c tnh theo cc php ton hng s cp (xem mt gio trnh bt k v i s tuyn tnh) Hai tnh cht quan trng ca nh thc l det Im = 1 v quy tc nhn det(AB) = det A det B. Mt ma trn thc K l c nghch o khi v ch khi nh thc ca n khc 0. Tuy nhin, iu quan trng cn nh l ta ang lm vic trn Z26. Kt qu tng ng l ma trn K c nghch o theo modulo 26 khi v ch khi UCLN(det K,26) = 1. Sau y s chng minh ngn gn kt qu ny. Trc tin, gi s rng UCLN(det K,26) = 1. Khi det K c nghch o trong Z26 . Vi 1 i m, 1 j m, nh ngha Ki j ma trn thu c t K bng cch loi b hng th i v ct th j. V nh ngha ma trn K* c phn t (i,j) ca n nhn gi tr(-1) det Kj i (K* c gi l ma trn b i s ca K). Khi c th chng t rng: K-1 = (det K)-1K* . Bi vy K l kh nghch. Ngc li K c nghch o K-1 . Theo quy tc nhn ca nh thc 1 = det I = det (KK-1) = det K det K-1 Bi vy det K c nghch o trong Z26 .
http://www.ebook.edu.vn 27

Nhn xt: Cng thc i vi trn khng phi l mt cng thc tnh ton c hiu qu tr cc trng hp m nh (chng hn m = 2, 3). Vi m ln, phng php thch hp tnh cc ma trn nghch o phi da vo cc php ton hng s cp. Trong trng hp 22, ta c cng thc sau: nh l Gi s A = (ai j) l mt ma trn cp 2 2 trn Z26 sao cho det A = a1,1a2,2 a1,2 a2,1 c nghch o. Khi
A = (det A)
-1 -1

a2,2 -a1,2 -a2,1 a1,1

Tr li v d xt trn . Trc ht ta c:
det 11 3 8 7 = 11 7 - 8 3 mod 2 = 77 - 24 mod 26 = 53 mod 26 =1

V 1-1 mod 26 = 1 nn ma trn nghch o l


11 3 8 7 -1 = 7 23 18 11

y chnh l ma trn c trn. By gi ta s m t chnh xc mt m Hill trn Z26 (hnh 1.6) Mt m HILL
Cho m l mt s nguyn dng c nh. Cho P = C = (Z26 )m v cho K = { cc ma trn kh nghch cp m m trn Z26} Vi mt kho K K ta xc nh eK(x) = xK v dK(y) = yK -1 Tt c cc php ton c thc hin trong Z26

2.1.6. Cc h m dng
http://www.ebook.edu.vn 28

Trong cc h mt nghin cu trn, cc phn t lin tip ca bn r u c m ho bng cng mt kho K. Tc xu bn m y nhn c c dng: y = y1y2. . . = eK(x1) eK(x2 ) . . . Cc h mt thuc dng ny thng c gi l cc m khi. Mt quan im s dng khc l mt m dng. tng c bn y l to ra mt dng kho z = z1z2 . . . v dng n m ho mt xu bn r x = x1x2 . . . theo quy tc: y = y1y2. . . = ez1(x1) ez2(x1). . . M dng hot ng nh sau. Gi s K K l kho v x = x1x2 . . .l xu bn r. Hm fi c dng to zi (zi l phn t th i ca dng kho) trong fi l mt hm ca kho K v i-1 l k t u tin ca bn r: zi = fi (K, x1 , . . ., xi -1 ) Phn t zi ca dng kho c dng m xi to ra yi = eiz(xi). Bi vy, m ho xu bn r x1 x2 . . . ta phi tnh lin tip: z1, y1, z2 , y2 ... Vic gii m xu bn m y1y2. . . c th c thc hin bng cch tnh lin tip: z1, x1, z2 , x2 ... Sau y l nh ngha di dng ton hc: nh ngha Mt m dng l mt b (P,C,K,L,F,E,D) tho mn dc cc iu kin sau: 1. 2. 3. 4. 5. P l mt tp hu hn cc bn r c th. C l tp hu hn cc bn m c th. K l tp hu hn cc kho c th ( khng gian kho) L l tp hu hn cc b ch ca dng kho. F = (f1 f2...) l b to dng kho. Vi i 1

fi : K P i -1 L 6. Vi mi z L c mt quy tc m ez E v mt quy tc gii m tng

ng dz D . ez : P C v dz : C P l cc hm tho mn dz(ez(x))= x vi mi bn r x P.

http://www.ebook.edu.vn

29

Ta c th coi m khi l mt trng hp c bit ca m dng trong dng kho khng i: Zi = K vi mi i 1. Sau y l mt s dng c bit ca m dng cng vi cc v d minh ho. M dng c gi l ng b nu dng kho khng ph thuc vo xu bn r, tc l nu dng kho c to ra ch l hm ca kho K. Khi ta coi K l mt "mn" m rng thnh dng kho z1z2 . . . Mt h m dng c gi l tun hon vi chu k d nu zi+d= zi vi s nguyn i 1. M Vigenre vi di t kho m c th coi l m dng tun hon vi chu k m. Trong trng hp ny, kho l K = (k1, . . . km ). Bn thn K s to m phn t u tin ca dng kho: zi = ki, 1 i m. Sau dng kho s t lp li. Nhn thy rng, trong m dng tng ng vi mt m Vigenre, cc hm m v gii m c dng ging nh cc hm m v gii m c dng trong MDV: ez(x) = x+z v dz(y) = y-z Cc m dng thng c m t trong cc b ch nhi phn tc l P= C=L= Z2. Trong trng hp ny, cc php ton m v gii m l php cng theo modulo 2. ez(x) = x +z mod 2 v dz(x) = y +z mod 2. Nu ta coi "0" biu th gi tr "sai" v "1" biu th gi tr "ng" trong i s Boolean th php cng theo moulo 2 s ng vi php hoc c loi tr. Bi vy php m (v gii m ) d dng thc hin bng mch cng. Ta xem xt mt phng php to mt dng kho (ng b) khc. Gi s bt u vi (k1, . . , km ) v zi = ki, 1 i m ( cng ging nh trc y), tuy nhin by gi ta to dng kho theo mt quan h quy tuyn tnh cp m: m-1 zi+m = cj zi+j mod j=0 trong c0, . . , cm-1 Z2 l cc hng s cho trc. Nhn xt:
http://www.ebook.edu.vn 30

Php quy c ni l c bc m v mi s hng ph thuc vo m s hng ng trc. Php quy ny l tuyn tnh bi v Zi+m l mt hm tuyn tnh ca cc s hng ng trc. Ch ta c th ly c0= 1 m khng lm mt tnh tng qut. Trong trng hp ngc li php quy s l c bc m-1. y kho K gm 2m gi tr k1, . . , km, c0, . . , cm-1. Nu (k1, . . , km)= (0,...,0) th dng kho s cha ton cc s 0. D nhin phi trnh iu ny v khi bn m s ng nht vi bn r. Tuy nhin nu chn thch hp cc hng s c0,..,cm-1 th mt vc t khi u bt k khc (k1, . . , km) s to nn mt dng kho c chu k 2m -1. Bi vy mt kho ngn s to nn mt dng kho c chu k rt ln. y l mt tnh cht rt ng lu tm v ta s thy phn sau, mt m Vigenre c th b thm nh tn dng yu t dng kho c chu k ngn. Sau y l mt v d minh ho: V d: Gi s m = 4 v dng kho c to bng quy tc: zi+4 = zi + zi+1 mod 2 Nu dng kho bt u mt vc t bt k khc vi vc t (0,0,0,0) th ta thu c dng kho c chu k 15. V d bt u bng vc t (1,0,0,0), dng kho s l: 1, 0, 0, 0, 1, 0, 0, 1, 1, 0, 1, 0, 1, 1, 1 Mt vc t khi u khc khng bt k khc s to mt hon v vng (cyclic) ca cng dng kho. Mt hng ng quan tm khc ca phng php to dng kho hiu qu bng phn cng l s dng b ghi dch hi tip tuyn tnh (hay LFSR). Ta dng mt b ghi dch c m tng. Vc t (k1, . . , km) s c dng khi to (t cc gi tr ban u) cho thanh ghi dch. mi n v thi gian, cc php ton sau s c thc hin ng thi. 1. 2. k1 c tnh ra dng lm bit tip theo ca dng kho. k2, . . , km s c dch mt tng v pha tri.
31

http://www.ebook.edu.vn

3. m-1

Gi tr mi ca ki s c tnh bng:

cjkj+1 j=0 (y l hi tip tuyn tnh) Ta thy rng thao tc tuyn tnh s c tin hnh bng cch ly tn hiu ra t mt s tng nht nh ca thanh ghi (c xc nh bi cc hng s cj c gi tr "1" ) v tnh tng theo modulo 2 ( l php hoc loi tr ). M t ca LFSR dng to dng kho Thanh ghi dch hi tip tuyn tnh (LFSR)

k1

k2

k3

k4

Mt v d v m dng khng ng b l m kho t sinh nh sau: (mt m ny do Vigenre xut). Mt m kho t sinh
Cho P = C = K = L = Z26 Cho z1 = K v zi = xi-1 (i 2) Vi 0 z 25 ta xc nh ez(x) = x + z mod 26 dz(y) = y - z mod 26 (x,y Z26 )

L do s dng thut ng "kho t sinh" l ch: bn r c dng lm kho (ngoi "kho khi thu" ban u K).
http://www.ebook.edu.vn 32

Sau y l mt v d minh ho Gi s kho l k = 8 v bn r l rendezvous. Trc tin ta bin i bn r thnh dy cc s nguyn: 17 4 13 3 4 25 21 14 20 18 Dng kho nh sau: 8 17 4 13 3 4 25 21 14 20 By gi ta cng cc phn t tng ng ri rt gn theo modulo 26: 25 21 17 16 7 3 20 9 8 12

Bn m dng k t l: ZVRQHDUJIM By gi ta xem Alice gii m bn m ny nh th no. Trc tin Alice bin i xu k t thnh dy s: 25 21 17 16 7 3 20 9 8 12

Sau c ta tnh: x1 = d8(25) = 25 - 8 mod 26 = 17 v x2 = d17(21) = 21 - 17 mod 26 = 4

v c tip tc nh vy. Mi khi Alice nhn c mt k t ca bn r, c ta s dng n lm phn t tip theo ca dng kho. D nhin l m dng kho t sinh l khng an ton do ch c 26 kho. Trong phn sau s tho lun cc phng php thm cc h mt m m ta trnh by. 2.2. M thm cc h m c in Trong phn ny ta s bn ti mt vi k thut m thm. Gi thit chung y l lun coi i phng Oscar bit h mt ang dng. Gi thit ny c gi l nguyn l Kerekhoff. D nhin, nu Oscar khng bit h mt c dng th nhim v ca anh ta s kh khn hn. Tuy nhin ta khng mun mt ca mt h mt li da trn mt gi thit khng chc chn l Oscar khng bit h

http://www.ebook.edu.vn

33

mt c s dng. Do , mc tiu trong thit k mt h mt l phi t c mt di gi thit Kerekhoff. Trc tin ta phn bit cc mc tn cng khc nhau vo cc h mt. Sau y l mt s loi thng dng nht. Ch c bn m: Thm m ch c xu bn m y. Bn r bit: Thm m c xu bn r x v xu bn m tng ng y. Bn r c la chn: Thm m nhn c quyn truy nhp tm thi vo c ch m ho. Bi vy, thm m c th chn mt xu bn r x v to nn xu bn m y tng ng. Bn m c la chn: Thm m c c quyn truy nhp tm thi vo c ch gii m. Bi vy thm m c th chn mt bn m y v to nn xu bn r x tng ng. Trong mi trng hp trn, i tng cn phi xc nh chnh l kho s dng. R rng l 4 mc tn cng trn c lit k theo tng ca sc mnh tn cng. Nhn thy rng, tn cng theo bn m c la chn l thch hp vi cc h mt kho cng khai m ta s ni ti chng sau. Trc tin, ta s xem xt cch tn cng yu nht, l tn cng ch c bn m. Gi s rng, xu bn r l mt vn bn ting Anh thng thng khng c chm cu hoc khong trng (m thm s kh khn hn nu m c du chm cu v khong trng). C nhiu k thut thm m s dng cc tnh cht thng k ca ngn ng ting Anh. Nhiu tc gi c lng tn s tng i ca 26 ch ci theo cc tnh ton thng k t nhiu tiu thuyt, tp ch v bo. Cc c lng trong bng di y ly theo ti liu ca Beker v Piper. Xc sut xut hin ca 26 ch ci:

http://www.ebook.edu.vn

34

K t sut A B C D E F G H I

Xc

K t sut

Xc

K t sut

Xc

.082 .015 .028 .043 .0127 .022 .020 .061 .070

J K L M N O P Q R

.002 .008 .040 .024 .067 .075 .019 .001 .060

S T U V W X Y Z

.063 .091 .028 .010 .023 .001 .020 .001

T bng trn, Beker v Piper phn 26 ch ci thnh 5 nhm nh sau: 1. 2. 3. 4. 0,023 5. V, K, J, X, Q, Z mi k t c xc sut nh hn 0,01 E: c xc sut khong 1,120 T, A, O, I, N, S, H, R : mi k t c xac sut khong 0,06 n 0,09 D, L : mi k t c xc sut chng 0,04 C, U, M, W, F, G, Y, P, B: mi k t c xc sut khong 0,015 n

Vic xem xt cc dy gm 2 hoc 3 k t lin tip (c gi l b idiagrams v b ba Trigrams) cng rt hu ch. 30 b i thng dng nht (theo th t gim dn) l: TH, HE, IN, ER, AN, RE, ED, ON, ES, ST, EN, AT, TO, NT, HA, ND, OU, EA, NG, AS, OR, TI, IS, ET, IT, AR, TE, SE, HI v OF. 12 b ba thng dng nht (theo th t gim dn) l: THE, ING, AND, HER, ERE, ENT, THA, NTH, WAS, ETH, FOR v DTH.
http://www.ebook.edu.vn 35

2.2.1. Thm h m Affine Mt m Affine l mt v d n gin cho ta thy cch thm h m nh dng cc s liu thng k. Gi s Oscar thu trm c bn m sau: Bng 1.2: Tn sut xut hin ca 26 ch ci ca bn m K t A B C D E F G sut 2 1 0 6 5 4 0 Tn t H I J K L M N K T n sut 5 0 0 5 2 2 1 t O P Q R S T K T n sut 1 3 0 8 3 0 t U V W X Y Z K T n sut 2 4 0 2 1 0

Bn m nhn c t m Affine: FMXVEDRAPHFERBNDKRXRSREFMORUDSDKDVSHVUFEDKPK DLYEVLRHHRH Phn tch tn sut ca bn m ny c cho bng di Bn m ch c 57 k t. Tuy nhin di ny cng phn tch thm m i vi h Affine. Cc k t c tn sut cao nht trong bn m l: R (8 ln xut hin), D (6 ln xut hin ), E, H, K (mi k t 5 ln ) v F, S, V ( mi k t 4 ln). Trong phng on ban u, ta gi thit rng R l k t m ca ch e v D l k t m ca t, v e v t tng ng l 2 ch ci thng dng nht. Biu th bng s ta c: eK(4) = 17 v eK(19) = 3. Nh li rng eK(x) = ax +b trong a v b l cc s cha bit. Bi vy ta c hai phng trnh tuyn tnh hai n: 4a +b = 17
http://www.ebook.edu.vn 36

19a + b = 3 H ny c duy nht nghim a = 6 v b = 19 ( trong Z26 ). Tuy nhin y l mt kho khng hp l do UCLN(a,26) = 2 1. Bi vy gi thit ca ta l khng ng. Phng on tip theo ca ta l: R l k t m ca e v E l m ca t. Thc hin nh trn, ta thu c a =13 v y cng l mt kho khng hp l. Bi vy ta phi th mt ln na: ta coi rng R l m ho ca e v H l m ho ca t. iu ny dn ti a = 8 v y cng l mt kho khng hp l. Tip tc, gi s rng R l m ho ca e v K l m ho ca t. Theo gi thit ny ta thu c a = 3 v b = 5 l kha hp l. Ta s tnh ton hm gii m ng vi K = (3,5) v gii m bn m xem liu c nhn c xu ting Anh c ngha hay khng. iu ny s khng nh tnh hp l ca kho (3,5). auk hi thc hin cc php ton ny, ta c dK (y) = 9y 19 v gii m bn m cho, ta c: algorithmsarequitegeneraldefinitionsof arithmeticprocesses Nh vy kho xc nh trn l kho ng. 2.2.2. Thm h m thay th Sau y ta phn tch mt tnh hung phc tp hn, l thay th bn m sau: V d: Bn m nhn c t MTT l: YIFQFMZRWQFYVECFMDZPCVMRZWNMDZVEJBTXCDDUMJ NDIFEFMDZCDMQZKCEYFCJMYRNCWJCSZREXCHZUNMXZ NZUCDRJXYMTMEYIFZWDYVZVYFZUMRZCRWNZDZJT XZWGCHSMRNMDHNCMFQCHZJMXJZWIEJYUCFWDINZDIR Phn tch tn sut ca bn m ny c cho bng di y: Tn sut xut hin ca 26 ch ci trong bn m.

http://www.ebook.edu.vn

37

K t A B C D E F G

T n sut 0 1 15 13 7 11 1 t

T n sut t

T n sut t

T n sut

H I J K L M N

4 5 11 1 0 16 9

O P Q R S T

0 1 4 10 3 2

U V W X Y Z

5 5 8 6 10 20

Do Z xut hin nhiu hn nhiu so vi bt k mt k t no khc trong bn m nn c th phng on rng, dZ(Z) = e. cc k t cn li xut hin t nht 10 ln ( mi k t ) l C, D, F, J, R, M, Y. Ta hy vng rng, cc k t ny l m kho ca (mt tp con trong) t, a, c, o, i, n, s, h, r, tuy nhin s khc bit v tn sut khng cho ta c c s phng on thch hp. Ti lc ny ta phi xem xt cc b i, c bit l cc b i c dng -Z hoc Z- do ta gi s rng Z s gii m thnh e. Nhn thy rng cc b i thng gp nht dng ny l DZ v ZW ( 4 ln mi b ); NZ v ZU ( 3 ln mi b ); v RZ, HZ, XZ, FZ, ZR, ZV, ZC, ZD v ZJ ( 2 ln mi b ). V ZW xut hin 4 ln cn WZ khng xut hin ln no v ni chung W xut hin t hn so vi nhiu k t khc, nn ta c th phng on l dK(W) = d. V DZ xut hin 4 ln v ZD xut hin 2 ln nn ta c th ngh rng dK(D) {r,s,t}, tuy nhin vn cn cha r l k t no trong 3 k t ny l k t ng. Nu tin hnh theo gi thit dK(Z) = e v dK(W) = d th ta phi nhn tr li bn m v thy rng c hai b ba ZRW v RZW xut hin gn u ca bn m

http://www.ebook.edu.vn

38

v RW xut hin li sau v R thng xut hin trong bn m v nd l mt b i thng gp nn ta nn th dK(R) = n xem l mt kh nng thch hp nht. Ti lc ny ta c: - - - - - - end - - - - - - - - - e - - - - ned- - - e - - - - - - - - YIFQFMZRWQFYVECFMDZPCVMRZWNMDZVEJBTXCDDUMJ - - - - - - - - e- - - - e - - - - - - - - n - - d - - - en - - - - e - - - -e NDIFEFMDZCDMQZKCEYFCJMYRNCWJCSZREXCHZUNMXZ - e - - - n - - - - - n - - - - - - ed - - - e - - - - - - ne - nd- e- e - NZUCDRJXYYSMRTMEYIFZWDYVZVYFZUMRZCRWNZDZJJ - ed - - - - - n - - - - - - - - - - e - - - ed - - - - - - - d - - - e - - n XZWGCHSMRNMDHNCMFQCHZJMXJZWIEJYUCFWDJNZDIR Bc tip theo l th dK(N) = h v NZ l mt b i thng gp cn ZN khng xut hin. Nu iu ny ng th on sau ca bn r ne - ndhe s gi rng dK(C) = a. Kt hp cc gi nh ny, ta c: - - - - - -end- - - - - a- - -e -a - - nedh- -e- - - - - -a - - - - YIFQFMZRWQFYVECFMDZPCVMRZWNMDZVEJBTXCDDUMJ h - - - - - - - a- - - e - a- - - a - - - nhad - a - -en -a - e - h- -e NDIFEFMDZCDMQZKCEYFCJMYRNCWJCSZREXCHZUNMXZ he - a - n- - - - - - n - - - - - - ed - - - e- - - e - - neandhe -e - NZUCDRJXYYSMRTMEYIFZWDYVZVYFZUMRZCRWNZDZJJ - ed - a - - -nh - - - ha - - - a- e - - - - ed - - - - -a -d - - he- -n XZWGCHSMRNMDHNCMFQCHZJMXJZWIEJYUCFWDJNZDIR By gi ta xt ti M l k t thng gp nht sau Z. on bn m RNM m ta tin l s gii m thnh nh- gi rng h- s bt u mt t, bi vy chc l M s biu th mt nguyn m. Ta s dng a v e, bi vy, phng on rng dK(M) = i hoc o. V ai l b i thng gp hn ao nn b i CM trong bn m gi rng, trc tin nn th dK(M) = i. Khi ta c:
http://www.ebook.edu.vn 39

- - - - -iend- - - - - a -i - e -a -inedhi - e- - - - - -a - - -i YIFQFMZRWQFYVECFMDZPCVMRZWNMDZVEJBTXCDDUMJ h - - - - - i - ea - i - e -a - - -a - i -nhad -a - en - -a - e -hi -e NDIFEFMDZCDMQZKCEYFCJMYRNCWJCSZREXCHZUNMXZ he - a - n - - - - -in -i - - - - ed - - -e - - - e - ineandhe - e - NZUCDRJXYYSMRTMEYIFZWDYVZVYFZUMRZCRWNZDZJJ - ed - a - - inhi - - hai - - a - e - i- -ed- - - - - a - d - - he - -n XZWGCHSMRNMDHNCMFQCHZJMXJZWIEJYUCFWDJNZDIR Tip theo th xc nh xem ch no c m ho thnh o. V o l mt ch thng gp nn gi nh rng ch ci tng ng trong bn m l mt trong cc k t D,F,J,Y. Y c v thch hp nht, nu khng ta s c cc xu di cc nguyn m, ch yu l aoi ( t CFM hoc CJM ). Bi vy gi thit rng dK(Y) = o. Ba k t thng gp nht cn li trong bn m l D,F,J, ta phn on s gii m thnh r,s,t theo th t no . Hai ln xut hin ca b ba NMD gi rng dK(D) = s ng vi b ba his trong bn r (iu ny ph hp vi gi nh trc kia l dK(D) {r,s,t} ). on HNCMF c th l bn m ca chair, iu ny s cho dK(F) = r (v dK(H) = c ) v bi vy (bng cch loi tr ) s c dK(J) = t. Ta c: o- r - riend - ro - - arise - a - inedhise - - t - - - ass - it YIFQFMZRWQFYVECFMDZPCVMRZNMDZVEJBTXCDDUMJ hs - r - riseasi - e - a - orationhadta - - en - -ace - hi - e NDIFEFMDZCDMQZKCEYFCJMYRNCWJCSZREZCHZUNMXZ he - asnt - oo - in - i - o - redso - e - ore - ineandhesett NZUCDRJXYYSMRTMEYIFZWDYVZVYFZUMRZCRWNZDZJJ - ed - ac - inhischair - aceti - ted - - to - ardsthes - n XZWGCHSMRNMDHNCMFQCHZJMXJZWIEJYUCFWDJNZDIR
http://www.ebook.edu.vn 40

By gi vic xc nh bn r v kho cho v d trn khng cn g kh khn na. Bn r hon chnh nh sau: Our friend from Pais examined his empty glass with surprise, as if evaporation had taen place while he wasn't looking. I poured some more wine and he settled back in his chair, face tilted up towards the sun. 2.2.3. Thm h m Vigenre Trong phn ny chng ta s m t mt s phng php thm h m Vigenre. Bc u tin l phi xc nh di t kho m ta k hiu l m. y dng hai k thut. K thut th nht l php th Kasiski v k thut th hai s dng ch s trng hp. Php th Kasiski ln u tin c Kasiski Friendrich m t vo nm 1863. K thut ny c xy dng trn nhn xt l: hai on ging nhau ca bn r s c m ho thnh cng mt bn m khi chng xut hin trong bn r cch nhau x v tr, trong x o mod m. Ngc li, nu ta thy hai on ging nhau ca bn m (mi on c di t nht l 3) th l mt du hiu tt ni rng chng tng ng vi cc on bn r ging nhau. Php th Kasiski nh sau. Ta tm trong bn m cc cp gm cc on nh nhau c di ti thiu l 3 v ghi li khong cch gia cc v tr bt u ca hai on. Nu thu c mt vi gi tr d1, d2 ,. . . th c th hy vng rng m s chia ht cho c chung ln nht ca cc di. Vic xc minh tip cho gi tr ca m c th nhn c bng ch s trng hp. Khi nim ny c Wolfe Friedman a ra vo 1920 nh sau: nh ngha: Gi s x = x1x2 . . . xn l mt xu k t. Ch s trng hp ca x (k hiu l Ic(x)) c nh ngha l xc sut hai phn t ngu nhin ca x l ng nht. Nu k hiu cc tn sut ca A,B,C,. . . ,Z trong x tng ng l f0,f1 ,. . . f25 , c th chn hai phn t ca x theo ??? cch. Vi mi i, 0 i 25, c ??? cch chn hai phn t l i.
http://www.ebook.edu.vn 41

By gi, gi s x l mt xu vn bn ting Anh. Ta k hiu cc xc sut xut hin ca cc k t A,B,. . .,Z trong bng 1.1 l p0,...p25. Khi : do xc sut hai phn t ngu nhin u l A l p02, xc sut c hai phn t ny u bng B bng p12 . . . Tnh hnh tng t cng xy ra nu x l mt bn m nhn c theo mt h m thay th n bt k. Trong trng hp ny, tng xc sut ring r s b hon v nhng tng ??? s khng thay i. By gi gi s c mt bn m y = y1y2. . .yn c cu trc theo mt m Vigenre. Ta xc nh cc xu con m ca y(y1,y2,. . .,ym) bng cch vit ra bn m thnh mt hnh ch nht c kch thc m(n/m). Cc hng ca ma trn ny l cc xu con yi, 1 i m. Nu m thc s l di kho th mi Ic(yi) phi xp x bng 0,065. Ngc li, nu m khng phi l di kho th cc xu con yi s c v ngu nhin hn v chng nhn c bng cch m dch vng vi cc kho khc nhau. Xt thy rng, mt xu hon ton ngu nhin s c: Hai gi tr 0,065 v 0,038 cch xa nhau c th xc nh c di t kho ng (hoc xc nhn gi thuyt c lm theo php th Kasiski). Hai k thut ny s c minh ho qua v d di y: V d: Bn m nhn c t mt m Vigenre. CHEEVOAHMAERATBTAXXWTNXBEEOPHBSBQMQEQERBW RVXUOAKXAOSXXWEAHBWGJMMQMNKGRFVGXWTRZXWIAK LXFPSKAUTEMNDCMGTSXMXBTUIADNGMGPSRELXNJELX RVPRTULHDNQWTWDTYGBPHXTFEALJHASVBFXNGLLCHR ZBWELEKMSJIKNBHWRJGNMGJSGLXFEYPHAGNRBIEQJT MRVLCRRREMNDGLXRRIMGNSNRWCHRQHAEYEVTAQEBBI EEWEVKAKOEWADREMXMTBHHCHRTKDNVRZCHRCLQOHP WQAIIWXNRMGWOIIFKEE Trc tin, ta hy th bng php th Kasiski xu bn m CHR xut hin bn v tr trong bn m, bt u cc v tr 1, 166,236 v 286. Khong cch t
http://www.ebook.edu.vn 42

ln xut hin u tin ti 3 ln xut hin cn li tng ng l 165,235 v 285. UCLN ca 3 s nguyn ny l 5, bi vy gi tr ny rt c th l di t kho. Ta hy xt xem liu vic tnh cc ch s trng hp c cho kt lun tng t khng. Vi m = 1 ch s trng hp l 0,045. Vi m = 2, c 2 ch s l 0,046 v 0,041. Vi m = 3 ta c 0,043; 0,050; 0,047. Vi m = 4 cc ch s l 0,042; 0,039; 0,046; 0,040. Vi m = 5 ta c cc gi tr 0,063; 0,068; 0,069; 0,061 v 0,072. iu ny cng chng t rng di t kho l 5. Vi gi thit trn, lm nh th no xc nh t kho? Ta s s dng khi nim ch s trng hp tng h ca hai xu sau: nh ngha. Gi s x = x1x2. . .xn v y = y1y2. . .yn' l cc xu c n v n' k t anphabet tng ng. Ch s trng hp tng h ca x v y ( k hiu l MIc(x,y)) c xc nh l xc sut mt phn t ngu nhin ca x ging vi mt phn t ngu nhin ca y. Nu ta k hiu cc tn sut ca A,B,. . .,Z trong x v y tng ng l f0,f1,. . .,f25 .Vi cc gi tr m xc nh, cc xu con yi thu c bng m dch vng bn r. Gi s K = (k1,k2,. . .,km) l t kho. Ta s xem xt c th nh gi MIc(yi,yj) nh th no. Xt mt k t ngu nhin trong yi v mt k t ngu nhin trong yj . Xc sut c hai k t l A bng p-ki p-kj, xc sut c hai l B bng p1-ki p1-kj,. . .(Cn ch rng tt c cc ch s di u c rt gn theo modulo 26). Bi vy c th c lng rng:
MI c ( y i , y i ) p h ki p h kj = p h p h + ki kj
h=0 h =0 25 25

Ta thy rng, gi tr c lng ny ch ph thuc vo kiu hiu ki-kj mod 26 (c gi l dch tng i ca yi v yj). Cng vy, ta thy rng:

p
h=0

25

h p h +1 = p h p h 1 h=0

25

Bi vy dch tng i l s dn n cng mt c lng MIc nh dch tng i 26-l .

http://www.ebook.edu.vn

43

Ta lp bng cc c lng cho dch tng i trong phm vi t 0 n 13.( Xem bng ). Cc ch s trng hp tng h tnh c.

tng i 0 1 2 3 4 5 6 7 8 9 10 11 12 13

dch MIc

Gi tr tnh c ca

0.065 0,039 0,032 0,034 0,044 0,033 0,036 0,039 0,034 0,034 0,038 0,045 0,039 0,043

Xt thy rng, nu dch tng i khc 0 th cc c lng ny thay i trong khong t 0.031 n 0,045; ngc li nu dch tng i bng 0 th c lng bng 0,065. C th dng nhn xt ny to nn mt phng on thch hp cho l = ki-kj ( dch tng i ca yi v yj) nh sau: Gi s c nh yi
http://www.ebook.edu.vn
g

44 25 MI c ( x , y ) =

f
i =0

f ' i g

n.n'

v xt vic m ho yj bng e0,e1,e2. . . Ta k hiu cc kt qu bng yj0,yj1,. . . D dng dng cc ch s MIc(yi,yjg), 0 g 25 theo cng thc sau: Khi g = l th MIc phi gn vi gi tr 0,065 v dch tng i ca yi v yj bng 0. Tuy nhin, vi cc gi tr g l th MIc s thay i gia 0,031 v 0,045. Bng k thut ny, c th thu c cc dch tng i ca hai xu con yi bt k. Vn cn li ch l 26 t kho c th v iu ny d dng tm c bng phng php tm kim vt cn. Tr li v d trn minh ho. trn gi nh rng, di t kho l 5. By gi ta s th tnh cc dch tng i. Nh my tnh, d dng tnh 260 gi tr MIc(yi,yjg), trong 1 i j 5; 0 g 25. Cc gi tr ny c cho trn bng. Vi mi cp ( i,j), ta tm cc gi tr ca MIc(yi,yjg) no gn vi 0,065. Nu c mt gi tr duy nht nh vy (i vi mi cp (i,j) cho trc), th c th phn on chnh l gi tr dch tng i. Trong bng di c 6 gi tr nh vy c ng khung. Chng chng t kh r rng l dch tng i ca y1 v y2 bng 9; dch tng i ca y2 v y3 bng 13; dch tng i ca y2 v y5 bng 7; dch tng i ca y3 v y5 bng 20; ca y4 v y5 bng 11. T y c cc phng trnh theo 5 n s K1, K2, K3, K4, K5 nh sau: K1 - K2 = 9 K1 - K2 = 16 K2 - K3 = 13 K2 - K5 = 17 K3 - K5 = 20 K4 - K5 = 11 iu ny cho php biu th cc Ki theo K1 ; K2 = K1 + 17
http://www.ebook.edu.vn 45

K3 = K1 + 4 K4 = K1 + 21 K5 = K1 + 10 Nh vy kho c kh nng l ( K1, K1+17, K1+4, K1+21, K1+10) vi gi tr K1 no Z26. T y ta hy vng rng, t kho l mt dch vng no ca AREVK. By gi, khng tn nhiu cng sc lm cng c th xc nh c t kho l JANET. Gii m bn m theo kho ny, ta thu c bn r sau: The almond tree was in tentative blossom. The days were longer often ending with magnificient evenings of corrugated pink skies. The hunting seasun was over, with hounds and guns put away for six months. The vineyards were busy again as the well-organized farmers treated their vinesand the more lackadaisical neighbors hurried to do the pruning they have done in November. . Cc ch s trng hp tng h quan st c. Gi tr ca MIc(yj,yjg)

0,028 0,027 0,028 0,034 0,039 0,037 0,026 0,025 0,052 0,068 0,044 0,026 0,037 0,043 0,037 0,043 0,037 0,028 0,041 0,041 0,041 0,034 0,037 0,051 0,045 0,042 0,036 0,039 0,033 0,040 0,034 0,028 0,053 0,048 0,033 0,029 0,056 0,050 0,045 0,039 0,040 0,036 0,037 0,032 0,027 0,037 0,047 0,032 0,027 0,039 0,037 0,039 0,035
http://www.ebook.edu.vn 46

0,034 0,043 0,025 0,027 0,038 0,049 0,040 0,032 0,029 0,034 0,039 0,044 0,044 0,034 0,039 0,045 0,044 0,037 0,055 0,047 0,032 0,027 0,039 0,037 0,039 0,035 0,043 0,033 0,028 0,046 0,043 0,044 0,039 0,031 0,026 0,030 0,036 0,040 0,041 0,024 0,019 0,048 0,070 0,044 0,028 0,038 0,044 0,043 0,047 0,033 0,026 0,046 0,048 0,041 0,032 0,036 0,035 0,036 0,020 0,024 0,039 0,034 0,029 0,040 0,067 0,061 0,033 0,037 0,045 0,033 0,033 0,027 0,033 0,045 0,052 0,042 0,030 0,046 0,034 0,043 0,044 0,034 0,031 0,040 0,045 0,040 0,048 0,044 0,033 0,024 0,028 0,042 0,039 0,026 0,034 0,050 0,035 0,032 0,040 0,056 0,043 0,028 0,028 0,033 0,033 0,036 0,046 0,026 0,018 0,043 0,080 0,050 0,029 0,031 0,045 0,039 0,037 0,027
http://www.ebook.edu.vn 47

0,026 0,031 0,039 0,040 0,037 0,041 0,046 0,045 0,043 0,035 0,030 0,038 0,036 0,040 0,033 0,036 0,060 0,035 0,041 0,029 0,058 0,035 0,035 0,034 0,053 0,030 0,032 0,035 0,036 0,036 0,028 0,043 0,032 0,051 0,032 0,034 0,030 0,035 0,038 0,034 0,036 0,030 0,043 0,043 0,050 0,025 0,041 0,051 0,050 0,035 0,032 0,033 0,033 0,052 0,031 0,027 0,030 0,072 0,035 0,034 0,032 0,043 0,027 0,052 0,038 0,033 0,038 0,041 0,043 0,037 0,048 0,028 0,028 0,036 0,061 0,033 0,033 0,032 0,052 0,034 0,027 0,039 0,043 0,033 0,027 0,030 0,039 0,048 0,035

2.2.4.Tn cng vi bn r bit trn h mt Hill. H m Hill l mt h mt kh pha hn nu tn cng ch vi bn m. Tuy nhin h mt ny d b ph nu tn cng bng bn r bit. Trc tin, gi s rng, thm m bit c gi tr m ang s dng. Gi s thm m c t nht m cp vc t khc nhau xj = (x1,j, x2,j, , . . ., xm,j) v yj = (y1,j, y2,j,...,ym,j) (1 j m)
http://www.ebook.edu.vn 48

sao cho yj = eK(xj), 1 j m. Nu xc nh hai ma trn: X = (xi,j) Y = (yi,j) cp mm th ta c phng trnh ma trn Y = XK, trong ma trn K cp mm l kho cha bit. Vi iu kin ma trn Y l kh nghch. Oscar c th tnh K = X1

Y v nh vy ph c h mt. (Nu Y khng kh nghch th cn phi th cc

tp khc gm m cp r - m). V d Gi s bn r Friday c m ho bng m Hill vi m = 2, bn m nhn c l PQCFKU. Ta c eK(5,17) = (15,16), eK(8,3) = (2,5) v eK(0,24) = (10,20). T hai cp r - m u tin, ta nhn c phng trnh ma trn:
15 16 5 2 5 = 8
17 K 3

Dng nh l d dng tnh c:


5 8
17 3
1

9 = 2

1 15

Bi vy:
9 K = 2
1 15 16 7 = 15 2 5 8 19 3

Ta c th dng cp r - m th 3 kim tra kt qu ny. Vn y l thm m phi lm g nu khng bit m?. Gi s rng m khng qu ln, khi thm m c th th vi m = 2,3,. . . cho ti khi tm c kho. Nu mt gi tr gi nh ca m khng ng th m trn mm tm c theo thut ton m t trn s khng tng thch vi cc cp r - m khc. Phng php ny, c th xc nh gi tr m nu cha bit. 2.2.5. Thm m h m dng xy dng trn LFSR. Ta nh li rng, bn m l tng theo modulo 2 ca bn r v dng kho, tc yi = xi + zi mod 2. Dng kha c to t (z1,z2,. . .,zm) theo quan h quy tuyn tnh:

http://www.ebook.edu.vn

49

z m +1 = c j z i +1 mod 2
j =0

m 1

trong c0,. . .,cm Z2 (v c0 = 1) V tt c cc php ton ny l tuyn tnh nn c th hy vng rng, h mt ny c th b ph theo phng php tn cng vi bn r bit nh trng hp mt m Hill. Gi s rng, Oscar c mt xu bn r x1x2. . .xn v xu bn m tng ng y1y2. . .yn . Sau anh ta tnh cc bt dng kho zi = xi+yi mod 2, 1 i n. Ta cng gi thit rng Oscar cng bit gi tr ca m. Khi Oscar ch cn tnh c0, . . ., cm-1 c th ti to li ton b dng kho. Ni cch khc, Oscar cn phi c kh nng xc nh cc gi tr ca m n s. Vi i 1 bt k ta c :
z m +1 = c j z i + j mod 2
j =0 m 1

l mt phng trnh tuyn tnh n n. Nu n 2n th c m phng trnh tuyn tnh m n c th gii c. H m phng trnh tuyn tnh c th vit di dng ma trn nh sau:
z 1 z2 . z z . (z m+1 , z m+2 ,...,z 2m ) = (c 0 , c1 ,...,c m1 ) 2 3 . . . zm zm+1 . . . . . . . . zm zm+1 . . z2m-1

Nu ma trn h s c nghch o ( theo modulo 2 )th ta nhn c nghim:


z 1 z2 . z z . (c 0 , c1 ,...,c m1 ) = (z m+1 , z m+2 ,...,z 2m ) 2 3 . . . zm zm+1 . . . . . . . . zm zm+1 . . z2m-1
1

Trn thc t, ma trn s c nghch o nu bc ca php quy c dng to dng kho l m.(xem bi tp). Minh ho iu ny qua mt v d. V d : Gi s Oscar thu c xu bn m 101101011110010
http://www.ebook.edu.vn 50

tng ng vi xu bn r 011001111111001 Khi anh ta c th tnh c cc bt ca dng kho: 110100100001010 Ta cng gi s rng, Oscar bit dng kho c to t mt thanh ghi dch phn hi (LFSR) c 5 tng. Khi , anh ta s gii phng trnh m trn sau ( nhn c t 10 bt u tin ca dng kho):
1 1 1 0 (0,1,0,0,0) = (c 0 , c1 , c 2 , c 3 , c 4 )0 1 1 0 0 0 0 1 0 0 1 1 0 0 1 0 0 0 1 0 0

C th kim tra c rng:


1 1 1 0 0 1 1 0 0 0 0 1 0 1 0 0 0 0 1 0 1 0 1 0 0
1

0 1 = 0 0 1

1 0 0 1 0

0 0 0 0 1

0 1 0 1 1

1 0 1 1 0

T ta c:
0 1 (c 0 , c1 , c 2 , c 3 , c 4 ) = (0,1,0,0,0)0 0 1 1 0 0 1 0 0 1 0 0 0 0 1 1 0 1 1 0 1 1 0

= (1, 0, 0, 1, 0) Nh vy php quy c dng to dng kho l: zi+5 = zi + zi+3 mod 2

http://www.ebook.edu.vn

51

Cc ch gii v ti liu dn Nhiu ti liu v mt m c in c trong cc gio trnh, chng hn nh gio trnh ca Beker v Piper [BP82] v Denning [DE82]. Xc sut nh gi cho 26 k t c ly ca Beker v Piper. Cng vy, vic phn tch m Vigenre c sa i theo m t ca Beker v Piper. Rosen [Ro93] l mt ti liu tham kho tt v l thuyt s. C s ca i s tuyn tnh s cp c th tm thy trong sch ca Anton [AN91]. Cun " Nhng ngi m thm " ca Kahn [KA67] l mt cu chuyn hp dn v phong ph v mt m cho ti nm 1967, trong Kahn khng nh rng mt m Vigenre thc s khng phi l pht minh ca Vigenre. Mt m Hill ln u tin c m t trong [HI29]. Cc thng tin v mt m dng c th tm c trong sch ca Rueppel [RU86].

http://www.ebook.edu.vn

52

Chng 3: Chun m d liu DES (Data Encryption Standard) 3.1. Gii thiu chung v DES Chun m ho d liu DES c Vn phng tiu chun ca M (U.S National Bureau for Standards) cng b nm 1971 s dng trong cc c quan chnh ph lin bang. Gii thut c pht trin ti Cng ty IBM da trn h m ho LUCIFER ca Feistel. DES l thut ton m ho khi (block algrithm), vi c ca mt khi l 64 bt. Mt khi 64 bt bn r c a vo, sau khi m ho d liu a ra l mt khi bn m 64 bt. C m ho v gii m u s dng cng mt thut ton v kho. Kho m c di 64 bt, trong c 8 bt chn l c s dng kim sot li. Cc bt chn l nm cc v tr 8, 16, 24,... , 64. Tc l c 8 bt kho th trong c 1 bt kim sot li, bt ny qui nh s bt c gi tr 1 ca khi 8 bt theo tnh b chn. Nn tng xy dng khi ca DES l s kt hp n gin ca cc k thut thay th v hon v bn r da trn kho. l cc vng lp. DES s dng 16 vng lp, n p dng cng mt kiu kt hp ca cc k thut trn khi bn r 16 ln (Nh hnh v) Thut ton ch s dng cc php ton s hc v lgc trn cc s 64 bt, v vy n d dng thc hin vo nhng nm 1970 trong iu kin v cng ngh phn cng lc by gi. Ban u, s thc hin cc phn mm kiu ny rt th s, nhng hin ti th vic tt hn, v vi c tnh lp i lp li ca thut ton to nn tng s dng chp vi mc ch c bit ny.

http://www.ebook.edu.vn

53

Plaintext IP L0 R0 K1 R1=L0(R0,K1) K2 R2=L1(R1,K2)

L1=R0

L2=R1

L15=R14

R15=L14(R14,K15) K16

R16=L15(R15,K16) IP-1 Ciphertext

L16=R15

S m DES

http://www.ebook.edu.vn

54

Tm li DES c mt s c im sau: S dng kho 56 bt. X l khi vo 64 bt, bin i khi vo thnh khi ra 64 bt. M ho v gii m c s dng cng mt kho. DES c thit k chy trn phn cng. DES thng c s dng m ho cc dng d liu mng v m ho d liu c lu tr trn a. 3.2. M t thut ton DES thc hin trn tng khi 64 bt bn r. Sau khi thc hin hon v khi u, khi d liu c chia lm hai na tri v phi, mi na 32 bt. Tip , c 16 vng lp ging ht nhau c thc hin, c gi l cc hm , trong d liu c kt hp vi kho. Sau 16 vng lp, hai na tri v phi c kt hp li v hon v cui cng (hon v ngc) s kt thc thut ton. Trong mi vng lp, cc bt ca kho c dch i v c 48 bt c chn ra t 56 bt ca kho. Na phi ca d liu c m rng thnh 48 bt bng mt php hon v m rng, tip khi 48 bt ny c kt hp vi khi 48 bt c thay i v hon v ca kho bng ton t XOR. Khi kt qu ca php tnh XOR c la chn ra 32 bt bng cch s dng thut ton thay th v hon v ln na. l bn thao tc to nn hm . Tip , u ra ca hm c kt hp vi na tri bng mt ton t XOR. Kt qu ca cc bc thc hin ny tr thnh na phi mi; na phi c tr thnh na tri mi. S thc hin ny c lp li 16 ln, to thnh 16 vng ca DES (Hnh 10). Nu Bi l kt qu ca vng th i, Li v Ri l hai na tri v phi ca Bi, Ki l kho 48 bt ca vng th i, v l hm thc hin thay th, hon v v XOR vi kho, ta c biu din ca mt vng s nh sau: Li=Ri-1 Ri=Li-1 XOR (Ri-1,Ki)

http://www.ebook.edu.vn

55

kha 28 bt Dch 28 bt 28 bt Dch 28 bt

56 bt Hon v Chn 48 bt

Ri-1 32 bt

M rng Hon v 48 bt

Hp S Thay th La chn 32 bt

Hp P Hon v

Ri

Li-1 32 btf

Li

Mt vng lp DES 3.3.Hon v khi u Hon v khi u i ch khi d liu vo, thay i v tr ca cc bt trong khi d liu vo, nh c m t trong Bng 1. Bng ny, v tt c cc bng khc sau ny, c c t tri qua phi, t trn xung di. V d, hon v khi u chuyn bt 1 thnh bt 58, bt 2 thnh bt 50, bt 3 thnh bt 42,... Bng 1. Hon v khi u.

http://www.ebook.edu.vn

56

58 50 42 34 26 18 10 2 62 54 46 38 30 22 14 6 57 49 41 33 25 17 9 1 61 53 45 37 29 21 13 5 n s an ton ca DES. 3.4. Kho chuyn i

60 52 44 36 28 20 12 4 64 56 48 40 32 24 16 8 59 51 43 35 27 19 11 3 63 55 47 39 31 23 15 7

Hon v khi u v tng ng l hon v ngc khng lm nh hng

u tin, kho 64 bt c gim xung thnh mt kho 56 bt bng cch b qua 8 bt chn l. S loi b c thc hin theo Bng sau: Bng kho chuyn i: 57 10 63 14 49 2 55 6 41 59 47 61 33 51 39 53 25 43 31 45 17 35 23 37 9 27 15 29 1 19 7 21 58 11 62 13 50 3 54 5 42 60 46 28 34 52 38 20 26 44 30 12 18 36 22 4

Cc bt chn l ny c th c s dng m bo rng khng c li no xy ra khi a kho vo. Sau khi kho 56 bt c trch ra, mt kho khc 48 bt c sinh ra cho mi vng ca DES. Nhng kho ny, ki, c xc nh bng cch: + u tin, kho 56 bt c chia lm hai phn mi phn 28 bt. Sau , cc phn ny c dch tri mt hoc hai bt, ph thuc vo vng . S bt c dch c cho trong Bng sau: Bng s bt dch ca mt vng Vng S bt dch 1 2 3 4 5 6 7 8 9 10 11 12 13 14 15 16 1 1 2 2 2 2 2 2 1 2 2 2 2 2 2 1

+ Sau khi c dch, 48 bt c la chn ra t 56 bt. Bi v s thc hin ny i ch th t cc bt nh l s la chn mt tp con cc bt, n c gi l hon v nn (compression permutation), hoc hon v la chn (permuted choice). S thc hin ny cung cp mt tp hp cc bt cng c vi u ra ca hon v m rng. Bng 4 nh ngha hon v nn (cng gi l hon http://www.ebook.edu.vn 57

v la chn). V d, bt v tr 33 ca kho dch c chuyn ti v tr 35 ca u ra, v bt v tr 18 ca kho dch b b qua. Bng hon v nn: 14 23 41 44 17 19 52 49 11 12 31 39 24 4 37 56 1 26 47 34 5 8 55 53 3 16 30 46 28 7 40 42 15 27 51 50 6 20 45 36 21 13 33 29 10 2 48 32

3.5. Hon v m rng thao tc ny, na phi ca d liu, Ri, c m rng t 32 bt thnh 48 bt. Bi v s thc hin ny thay i th t ca cc bt bng cch lp li mt bt no , n c hiu nh l mt s hon v m rng. S thc hin ny nhm mc ch to ra kt qu l d liu cng c vi kho thc hin thao tc XOR. nh ngha hon v m rng - hp E. Vi mi b 4 bt ca khi d liu vo, bt u tin v bt th t mi bt tng ng vi 2 bt ca khi d liu ra, trong khi bt th hai v bt th ba mi bt tng ng vi mt bt ca khi d liu ra. Bng di m t v tr ca cc bt trong khi d liu ra theo khi d liu vo. V d, bt v tr th 3 ca khi d liu vo c chuyn ti v tr th 4 trong khi d liu ra. V bt v tr 21 ca khi d liu vo c chuyn ti v tr 30 v 32 trong khi d liu ra. Bng hon v m rng E: 32 8 16 24 1 9 17 25 2 10 18 26 3 11 19 27 4 12 20 28 5 12 21 29 4 12 20 28 5 13 21 29 6 14 22 30 7 15 23 31 8 16 24 32 9 17 25 1

http://www.ebook.edu.vn

58

2 3 4

6 7 8

9 10 11 12

13 14 15 16

32

48

1 2 3

4 5 6

7 8 9 10 11 12

13 14 15 16 17 18

19 20 21 22 23 24

Hon v m rng Mc d khi d liu ra rng hn khi d liu vo, nhng mt khi d liu vo ch c duy nht mt khi d liu ra. 3.6. Hp thay th S Sau khi c nn, kho c XOR vi khi m rng, 48 bt kt qu c chuyn sang giai on thay th. S thay th c thc hin bi 8 hp thay th (substitution boxes, S-boxes). Khi 48 bt c chia thnh 8 khi 6 bt. Mi khi c thc hin trn mt hp S ring bit (separate S-box): khi 1 c thc hin trn hp S1, khi 2 c thc hin trn hp S2,... , khi 8 c thc hin trn hp S8. http://www.ebook.edu.vn 59

Mi hp S l mt bng gm 4 hng v 16 ct. Mi phn t ca hp l mt s 4 bt. Su bt vo hp S s xc nh s hng v s ct tm kt qu ra. Bng 6 biu din 8 hp S. Nhng bt vo xc nh mt phn t trong hp S mt cch ring bit. Su bt vo ca hp c k hiu l b1, b2, b3, b4, b5 v b6. Bt b1 v b6 c kt hp thnh mt s 2 bt, nhn gi tr t 0 n 3, tng ng vi mt hng trong bng. Bn bt gia, t b2 ti b5, c kt hp thnh mt s 4 bt, nhn gi tr t 0 n 15, tng ng vi mt ct trong bng. V d, gi s ta a d liu vo hp S th 6 (bt 31 ti bt 36 ca hm XOR) l 110010. Bt u tin v bt cui cng kt hp thnh 10, tng ng vi hng th 3 ca hp S th 6. Bn bt gia kt hp thnh 1001, tng ng vi ct th 10 ca hp S th 6. Phn t hng 3 ct 9 ca hp S th 6 l 0. Gi tr 0000 c thay th cho 110010. Kt qu ca s thay th l 8 khi 4 bt, v chng c kt hp li thnh mt khi 32 bt. Khi ny c chuyn ti bc tip theo: hp hon v P (Pbox permutation). Hp S th nht 14 4 0 4 1 13 1 4 2 14 6 7 14 8 2 15 11 8 13 1 2 1 4 7 9 5 3 10 6 12 5 7 3 9 5 0 3 6 5 2 7 8 0 13 10 15 15 7 14 2 13 6 4 9 11 3 8 10 6 5 7 8 12 11 9

11 15 12 9 11 3 2 1 7

10 5

15 12 8 15 1 3 0 8

14 10 0 13 12 0 10 6 9 12 0 9 3 5

Hp S th 2 13 4 14 7 15 2 3 14 12 0 2 11 6 11 5 14 9

11 10 4

13 1

12 6

13 8

10 1 Hp S th 3

15 4

http://www.ebook.edu.vn

60

10 0 13 7 13 6 1 7

9 0 4

14 6 9 9 3 8 6 0 6

3 4 9 6

15 5 6 8 9 0 7

13 12 7 8 5 2

11 4

10 2 4

14 12 11 15 1 12 5 10 14 7 2 12 15 4 14 11 5

15 3

11 1

10 13 0 Hp S th 4 13 14 3 11 5 9 0 6

15 14 3 2 7 4 8 2 3 5 5

10 1 3 4 9

11 12 4 2 8 2

13 8 10 6 3

15 0

12 1 14 5

10 14 9

12 11 7 10 1

13 15 1

15 0

13 8

11 12 7

Hp S th 5 2 4 12 4 2 1 1 7 10 11 6 7 13 1 8 8 5 5 0 3 15 13 0 9 3 6 14 9 8 0 5 5 6 14 3 11 8 13 1 6 2 12 14 11 2 11 8 12 1 9 4 4 1 6 12 4 1 15 10 3 12 5 9 4

11 10 13 7 14 2 2 8 5 6

15 9

12 7

13 6 8 5 0 6 7

15 0 13 3 1 0 4

10 4 14 7

Hp S th 6 10 15 9 2 7 2 10 15 4 3 2 12 9 13 14 0 10 1 7 7 8 6 5 0 11 3 0 8

14 15 5 Hp S th 7 11 2 4

12 3

13 11 6

12 9

15 10 11 14 1 8 1 7 13 3 12 9 5 0

14 15 0 4 1 9 4

10 6 15 8 5 9 3

13 0

11 7

10 14 3 9 5

12 2

11 13 12 3

14 10 15 6

11 13 8 Hp S th 8

10 7

15 14 2

http://www.ebook.edu.vn

61

13 2 1 7 2

4 1

6 9 4

15 11 1 7 4 12 14 2 10 8

10 9 12 5 0 6

3 6

14 5 11 0 0 3

12 7 2 8 11 5 6

15 13 8 11 4 1 14 7

10 3

14 9 5

10 13 15 3

13 15 12 9

3.7. Hp hon v P Khi d liu 32 bt ra ca hp thay th S c hon v tip trong hp P. S hon v ny nh x mi bt d liu vo ti mt v tr trong khi d liu ra; khng bt no c s dng hai ln v cng khng bt no b b qua. N c gi l hon v trc tip (straight permutation). Bng hon v cho ta v tr ca mi b cn chuyn. V d, bt 4 chuyn ti bt 21, trong khi bt 32 chuyn ti bt 4. Bng hp hon v P 16 7 2 8 20 21 29 12 28 17 1 24 14 32 27 3 9 15 23 26 5 18 31 10 25 19 13 30 6 22 11 4

Cui cng, kt qu ca hp ho v P c XOR vi na tri ca khi 64 bt khi u. Sau , na tri v phi c chuyn i cho nhau v mt vng mi c tip tc. 3.8. Hon v cui cng Hon v cui cng l nghch o ca hon v khi u, v n c m t trong bng di. Ch rng na tri v na phi khng c tro i sau vng cui cng ca DES; thay vo khi ni R16L16 c s dng nh khi d liu ra ca hon v cui cng. Khng c g a ra y; tro i cc na v dch vng hon v s cho chnh xc nh kt qu trc; iu c ngha l thut ton c th c s dng cho c m ho v gii m. Bng hon v cui cng: 40 8 38 6 36 4 48 16 56 24 64 32 39 7 46 14 54 22 62 30 37 5 44 12 52 20 60 28 35 3 62 47 15 55 23 63 31 45 13 53 21 61 29 43 11 51 19 59 27

http://www.ebook.edu.vn

34 2

42 10 50 18 58 26 33 1 3.9. Gii m DES

41 9

49 17 57 25

Sau khi thay i, hon v, XOR, v dch vng, chng ta c th ngh rng thut ton gii m phc tp, kh hiu nh thut ton m ho v hon ton khc thut ton m ho. Tri li, s hot ng c la chn a ra mt c tnh hu ch: cng thut ton lm vic cho c m ho v gii m. Vi DES, c th s dng cng chc nng gii m hoc m ho mt khi. Ch c s khc nhau l cc kho phi c s dng theo th t ngc li. Ngha l, nu cc kho m ho cho mi vng l k1, k2, k3 ,... , k15, k16 th cc kho gii l k16, k15,... , k3, k2, k1. Gii thut tng hp kho cho mi vng cng tng t. C khc l cc kho c dch phi v s v tr bit dch c ly theo chiu ngc li. 3.10. Phn cng v phn mm thc hin DES Vic m t DES kh di dng song vic thc hin DES rt hu hiu bng c phn cng ln phn mm. Cc php tnh s hc duy nht c thc hin l php XOR cc xu bt. Hm m rng E, cc hp S, cc hon v khi u IP, hon v cui cng IP-1 v vic tnh ton cc kho k1, k2,... , k16 u c th thc hin c cng lc bng tra bng (trong phn mm) hoc bng cch ni cng chng thnh mch. Mt phn mm DES trn my tnh ln IBM 3090 c th thc hin 32.000 php tnh m ho trong mt giy. Vi my vi tnh th tc thp hn. Bng 9 a ra kt qu thc t v s nh gi cho b x l ca Intel v Motorola. Bng 9. Tc ca DES trn cc b vi x l khc nhau Tc B vi x l 8088 68000 80286 ( Mhz ) 4.7 7.6 6.0 BUS ( bt ) 8 16 16 63 Khi DES (/giy) 370 900 1.100

http://www.ebook.edu.vn

68020 68030 80286 68030 68040 68040 80486

16.0 16.0 25.0 50.0 25.0 40.0 33.0

32 32 16 32 32 32 32

3.500 3.900 5.000 9.600 16.000 23.200 40.600

(Ch : Phn mm ny c vit trn C v Assembler, v c th mua c t Utimaco-Belgium, Interleuvenlaan 62A, B-300 leuven, Belgium. C m xp x 64K. ANSI C thc hin chm hn khong 20%.) Mt ng dng rt quan trng ca DES l trong giao dch ngn hng M. DES c dng m ho cc s nh danh cc nhn (PIN) v vic chuyn ti khon c thc hin bng my th qu t ng (ATM). DES cn c s dng rng di trong cc t chc chnh ph. 3.11. S an ton ca DES c rt nhiu s nghin cu v di ca kho, s vng lp, v thit k ca hp S (S-boxes). Hp S c c im l kh hiu, khng c bt c s r ang no nh ti sao chng phi nh vy. Mi tnh ton trong DES ngoi tr cc hp S u tuyn tnh, tc vic tnh XOR ca hai u ra cng ging nh php XOR hai u vo ri tnh ton u ra. Cc hp S cha ng thnh phn phi tuyn ca h l yu t quan trng nht i vi s an ton ca h thng. Tnh bo mt ca mt h m ho i xng l mt hm hai tham s: phc tp ca thut ton v di ca kho. Gi s rng tnh bo mt ch ph thuc vo phc tp ca thut ton. C ngha rng s khng c phng php no ph v h thng mt m hn l c gng th mi kho c th, phng php c gi l brute-force attack. Nu kho c di 8 bt, suy ra s c 28=256 kho. V vy, s mt nhiu nht 256 ln th tm ra kho ng. Nu kho c di 56 bt, th s c 256 kho c th s dng. Gi s mt Suppercomputer c th th mt triu kho trong mt giy, th n s cn 2000 nm tm ra kho ng. Nu kho http://www.ebook.edu.vn 64

c di 64 bt, th vi chic my trn s cn 600,000 nm tm ra kho ng trong s 264 kho. Nu kho c di 128 bt, th s mt 1025 nm tm ra kho ng. V tr ch mi tn ti 1010 nm, v vy 1025 th mt thi gian qu di. Vi mt kho 2048 bt, mt my tnh song song thc hin hng t t php th trong mt giy s tiu tn mt khong thi gian l 10597 nm tm ra kho. Lc v tr c l khng cn tn ti na. Khi IBM a ra thit k u tin ca h m ho LUCIFER, n c kho di 128 bt. Ngy nay, DES tr thnh mt chun v m ho d liu s dng kho 56 bt, tc kch thc khng gian kho l 256. Rt nhiu nh m ho hin ang tranh lun v mt kho di hn ca DES. Nhiu thit b chuyn dng c xut nhm phc v cho vic tn cng DES vi bn r bit. S tn cng ny ch yu thc hin tm kho theo phng php vt cn. Tc vi bn r X 64 bt v bn m Y tng ng, mi kho c th u c kim tra cho ti khi tm c mt kho k tho mn Ek(X)=Y (c th c nhiu hn mt kho k nh vy). Vo nm 1979, Diffie v Hellman tuyn b rng vi mt my tnh chuyn dng bn m ho DES c th c ph bng cch th mi trng hp ca kho trong vng mt ngy gi ca my tnh l 20 triu la. Vo nm 1981, Diffie tng ln l cn hai ngy tm kim v gi ca chic my tnh l 50 triu la. 3.12. Tranh lun v DES. Khi DES c xut nh mt chun mt m, c rt nhiu kin ph phn. Mt l do phn i DES c lin quan n cc hp S. Mi tnh ton lin quan n DES ngoi tr cc hp S u tuyn tnh, tc vic tnh php hoc loi tr ca hai u ra cng ging nh php hoc loi tr ca hai u vo ri tnh tan u ra. Cc hp S cha ng thnh phn phi tuyn ca h mt l yu t quan trong nht i vi mt ca h thng( Ta thy trong chng 1 l cc h mt tuyn tnh chng hn nh Hill c th d dng b m thm khi b tn cng bng bn r bit). Tuy nhin tiu chun xy dng cc hp S khng c bit y . Mt s ngi gi l cc hp S phi cha cc http://www.ebook.edu.vn 65

ca sp c du kn, cho php Cc An ninh Quc gia M (NSA) gii m c cc thng bo nhng vn gi c mc an ton ca DES. D nhin ta khng th bc b c khng nh ny, tuy nhin khng c mt chng c no c a ra chng t rng trong thc t c cc ca sp nh vy. Nm 1976 NSA khng nh rng, cc tnh cht sau ca hp S l tiu chun thit k: P0 Mi hng trong mi hp S l mt hon v ca cc s nguyn 0, 1, . . . , 15. P1 Khng mt hp S no l mt hm Affine hoc tuyn tnh cc u vo ca n. P2 Vic thay i mt bt vo ca S phi to nn s thay i t nht l hai bt ra. P3 i vi hp S bt k v vi u vo x bt k S(x) v S(x 001100) phi khc nhau ti thiu l hai bt ( trong x l xu bt di 6 ). Hai tnh cht khc nhau sau y ca cc hp S c th coi l c rt ra t tiu chun thit k ca NSA. P4 Vi hp S bt k, u vo x bt k v vi e, f {0,1}: S(x) S(x 11ef00). P5 Vi hp S bt k , nu c nh mt bt vo v xem xt gi tr ca mt bt u ra c nh th cc mu vo bt ra ny bng 0 s xp x bng s mu ra bt bng 1.( Ch rng, nu c nh gi tr bt vo th nht hoc bt vo th 6 th c 16 mu vo lm cho mt bt ra c th bng 0 v c 16 mu vo lm cho bt ny bng 1. Vi cc bt vo t bt th hai n bt th 5 th iu ny khng cn ng na. Tuy nhin phn b kt qu vn gn vi phn b u. Chnh xc hn, vi mt hp S bt k, nu ta c nh gi tr ca mt bt vo bt k th s mu vo lm cho mt bt ra c nh no c gi tr 0 (hoc 1) lun nm trong khong t 13 n 19). Ngi ta khng bit r l liu c cn mt chun thit k no y hn c dng trong vic xy dng hp S hay khng. S phn i xc ng nht v DES chnh l kch thc ca khng gian kho: 256 l qu nh m bo an ton thc s. Nhiu thit bi chuyn dng c xut nhm phc v cho vic tn cng vi bn r bit. Php tn cng ny ch yu thc hin tm kho theo phng php vt cn. Tc vi bn r x 64 bt v bn m y tng ng, mi kho u c th c kim tra cho ti http://www.ebook.edu.vn 66

khi tm c mt kho K tho mn eK(x) = y. Cn ch l c th c nhiu hn mt kho K nh vy). Ngay t nm 1977, Diffie v Hellman gi rng c th xy dng mt chp VLSI (mch tch hp mt ln) c kh nng kim tra c 106kho/giy. Mt my c th tm ton b khng gian kho c 106 trong khong 1 ngy. H c tnh chi ph to mt my nh vy khong 2.107$. Trong cuc hi tho ti hi ngh CRYPTO93, Michael Wiener a ra mt thit k rt c th v my tm kho. My ny c kh nng thc hin ng thi 16 php m v tc ti 5107 kho/giy. Vi cng ngh hin nay, chi ph ch to khong 10,5$/khung. Gi ca mt khung my cha 5760 chp vo khong 100.000$ v nh vy n c kh nng tm ra mt kho ca DES trong khong 1,5 ngy. Mt thit b khung 10 khung my nh vy c gi chng 106 $ s gim thi gian tm kim kho trng bnh xung cn 3,5 gi. 3.13. DES trong thc t. Mc d vic m t DES kh di dng song ngi ta c th thc hin DES rt ha hiu bng c phn cng ln phn mn. Cc php ton duy nht cn c thc hin l php hoc loi tr cc xu bt. Hm m rng E, cc hp S, cc hon v IP v P v vic tnh ton cc gi tri K1,.. . ,K16 u c th thc hin c cng lc bng tra bng (trong phn mn) hoc bng cch ni cng chng thnh mt mch. Cc ng dng phn cng hin thi c th t c tc m ho cc nhanh. Cng ty Digital Equipment thng bo ti hi ngh CRUPTO92 rng h s ch to mt xung c 50 ngn xung c th m ho vi tc 1 Gbt/s bng cch xung nhp c tc 250MHz. Gi ca xung ny vo khong 300$. Ti nm 1991 c 45 ng dng phn cng v chng trnh c s ca DES c U ban tiu Chun quc gia M (NBS) chp thun. Mt ng dng quan trng ca DES l trong giao dch ngn hng M (ABA) DES c dng m ho cc s nh danh c nhn (PIN) v vic chuyn ti khon bng my th qu t ng (ATM). DES cng c H thng chi tr gia cc nh bng ca Ngn hng hi oi (CHIPS) dng xc http://www.ebook.edu.vn 67

thc cc giao dch vo khon trn 1,51012 USA/tun. DES cn c s dng rng ri trong cc t chc chnh ph. Chng hn nh b nng lng, B T php v H thng d tr lin bang. 3.14. Cc ch hot ng ca DES. C 4 ch lm vic c pht trin cho DES: Ch chuyn m in t (ECB), ch phn hi m (CFB), ch lin kt khi m (CBC) v ch phn hi u ra (OFB). Ch ECB tng ng vi cch dng thng thng ca m khi: vi mt dy cc khi bn r cho trc x1,x2,. . .( mi khi c 64 bt), mi xi s c m ho bng cng mt kho K to thnh mt chui cc khi bn m y1y2 ... theo quy tc yi = eK(yi-1xi) i 1. Vic s dng ch CBC c m t trn hnh 3.4. Hnh 3.4. Ch CBC.

http://www.ebook.edu.vn

68

x1

x2

IV=y0 M ho Encrypt

...

eK

eK

y1

y2

y1 Gii m Decrypt

y2

dK

dK

IV=y0

...

x1

x2

Trong cc ch OFB v CFB dng kho c to ra s c cng mod 2 vi bn r (tc l n hot ng nh mt h m dng, xem phn 1.1.7). OFB thc s l mt h m dng ng b: dng kho c to bi vic m lp vc t khi to 64 bt (vc t IV). Ta xc nh z0 =IV v ri tnh dng kho z1z2 . . . theo quy tc zi = eK(zi-1), i1. Dy bn r x1x2 . . . sau s c m ho bng cch tnh yi = xi zi,i 1. Trong ch CFB, ta bt u vi y0 = IV (l mt vc t khi to 64 bt) v to phn t zi ca dng kho bng cch m ho khi bn m trc . Tc zi = eK(yi-1), i 1. Cng nh trong ch OFB: yi = xi zi,i 1. Vic s dng CFB c m t trn hnh 3.5 (ch rng hm m DES eK c dng cho c php m v php gii m cc ch CFB v OFB). Hnh 3.5. Ch CFB http://www.ebook.edu.vn 69

x1

x2

IV=y0

eK

eK

...

M ho Encrypt

y1

y2

y1

y2

IV=y0

eK

eK

...

Gii m Decrypt

x1

x2

Cng cn mt s bin tu ca OFB v CFB c gi l cc ch phn hi K bt (1 < K < 64 ). y ta m t cc ch phn hi 64 bt. Cc ch phn hi 1 bt v 8 bt thng c dng trong thc t cho php m ho ng thi 1 bit (hoc byte) s liu. Bn ch cng tc c nhng u, nhc im khc nhau. ch ECB v OFB, s thay i ca mt khi bn r xi 64 bt s lm thay i khi bn m yi tng ng, nhng cc khi bn m khc khng b nh hng. http://www.ebook.edu.vn 70

Trong mt s tnh hung y l mt tnh cht ng mong mun. V d, ch OFB thng c dng m khi truyn v tinh. Mt khc cc ch CBC v CFB, nu mt khi bn r xi b thay i th yi v tt c cc khi bn m tip theo s bi nh hng. Nh vy cc ch CBC v CFB c th c s dng rt hiu qu cho mc ch xc thc. c bit hn, cc ch ny c th c dng to m xc thc bn tin ( MAC message authentication code). MAC c gn thm vo cc khi bn r thuyt phc Bob tin rng, dy bn r thc s l ca Alice m khng b Oscar gi mo. Nh vy MAC m bo tnh ton vn (hay tnh xc thc) ca mt bn tin ( nhng tt nhin l MAC khng m bo mt). Ta s m t cchb s dng ch BCB to ra mt MAC. Ta bt u bng vc t khi t IV cha ton s 0. Sau dng ch CBC to cc khi bn m y1,. . . ,yn theo kho K. Cui cng ta xc nh MAC l yn. Alice s pht i dy cc khi bn r x1,x2,. . . ,xn cng vi MAC. Khi Bob thu c x1. . .xn anh ta s khi phc li y1. . .yn bng kho K b mt v xc minh xem liu yn c ging vi MAC m mnh thu c hay khng. Nhn thy Oscar khng th to ra mt MAC hp l do anh ta khng bit kho K m Alice v Bob ang dng. Hn na Oscar thu chn c dy khi bn r x1. . .xn v thay i t nhiu ni dung th th chc chn l Oscar khng th thay i MAC c Bob chp nhn. Thng thng ta mun kt hp c tnh xc thc ln bo mt. iu c th thc hin nh sau: Trc tin Alice dng kho K1 to MAC cho x1. . . xn . Sau Alice xc nh xn+1 l MAC ri m ho dy x1. . .xn+1 bng kho th hai K2 to ra bn m y1. . .yn+1 . Khi Bob thu c y1. . .yn+1 , trc tin Bob s gii m ( bng K2) v kim tra xem xn+1 c phi l MAC i vi dy x1. . .xn dng K1 hay khng. Ngc li, Alice c th dng K1 m ho x1. . .xn v to ra c y1...yn , sau dng K2 to MAC yn+1 i vi dy y1. . .yn. Bob s dng K2 xc minh MAC v dung K1 gii m y1. . .yn

http://www.ebook.edu.vn

71

Chng 4: Mt m cng khai 4.1. Gii thiu v h mt m kha cng khai. 4.1.1. Gii thiu. Trong m hnh mt m c in m cho ti nay vn cn ang c nghin cu Alice (ngi gi) v Bob (ngi nhn) bng cch chn mt kho b mt K. Sau Alice dng kho K m ho theo lut eK v Bod dng kho K gii m theo lut gii dK . Trong h mt ny, dK hoc ging nh eK hoc d dng nhn c t n v qu trnh gii m hon ton tng t nh qu trnh m, nhng th tc kho th ngc li. Nhc im ln ca h mt ny l nu ta l eK th lm cho h thng mt an ton, chnh v vy chng ta phi to cho cc h mt ny mt knh an ton m kinh ph to mt knh an ton khng phi l r. tng xy dng mt h mt kho cng khai l tm mt h mt khng c kh nng tnh ton xc nh dK nu bit c eK. Nu thc hin c nh vy th quy tc m eK c th c cng khai bng cch cng b n trong danh b, v khi Alice (ngi gi) hoc bt c mt ai mun gi mt bn tin cho Bob (ngi nhn) th ngi khng phi thng tin trc vi Bob (ngi nhn) v kho mt, m ngi gi s m ho bn tin bng cch dng lut m cng khai eK. Khi bn tin ny c chuyn cho Bob (ngi nhn) th ch c duy nht Bob mi c th gii c bn tin ny bng cch s dng lut gii m b mt dK. tng v h mt kho cng khai c Diffie v Heliman a ra vo nm 1976. Cn vic thc hin h mt kho cng khai th li c Rivest. Shamin v Adieman a ra u tin vo nm 1977. H to nn h mt RSA ni ting. K t c mt s h mt c cng b, mt ca tng h da trn cc bi ton tnh ton khc nhau. Trong quan trng nht l cc h mt sau: H mt RSA bo mt ca h RSA da trn kh ca vic phn tch ra tha s nguyn t cc s nguyn t ln. http://www.ebook.edu.vn 72

H mt xp bal Merkle Hellman. H ny v cc h c lin quan da trn tnh kh gii ca bi ton tng cc tp con. H mt McEliece H mt nan da trn l thuyt m i s v vn c coi l an ton. H mt McEliece da trn bi ton gii m cho cc m tuyn tnh. H mt ElGamal H ElGamal da trn tnh kh gii ca bi ton Logarit ri rc trn cc trng hu hn. H mt Chor Rivest H mt Chor Rivest cng c xem nh mt loi h mt xp bal. Tuy nhin h mt ny vn cn c coi l h mt an ton. H mt trn cc ng cong Elliptic. Cc h ny l bin tng ca h mt khc, chng lm vic trn cc ng cong Elliptic ch khng phi trn cc trng hu hn. H mt ny m bo mt v kho s nh hn cc h mt kho cng khai khc. Mt ch quan trng l mt h mt kho cng khai khng bao gi c th bo m c mt tuyt i (an ton v in kin). S d vy v i phng nghin cu mt bn m C c th m ln lt cc bn r c th bng lut m cng khai eK cho ti khi anh ta tm c mt bn r duy nht P bo m C = eK(P). Bn r P ny chnh l kt qu gii m ca C. Bi vy ta ch nghin cu mt v mt tnh ton ca h ny. Mt ch quan trng v c ch khi nghin cu na l khi nim v hm ca sp mt chiu. Ta nh ngha khi nim ny mt cch khng hnh thc. nh ngha: Hm f: X Y c gi l hm mt chiu nu tnh y=f(x) vi mi x X l d nhng vic tm x khi bit y li l vn kh. Thc ra pht biu trn ch l nh ngha phi hnh thc (do thut ng kh c dng n l khng nh lng v thm ch sau ny chng ta bit l ngay c khi nh lng bng s khng tn ti thut ton gii bi http://www.ebook.edu.vn 73

ton ngc trong phm vi a thc th khi nim kh nu trn c tn ti hay khng cng cha c ai khng nh r rng) v iu ng tic hn na l tt c cc hm ng c vin cho khi nim ny cho n nay ch mi c coi l mt chiu. Chng ta d dng thng nht c vi nhau l ch ring hm mt chiu l khng xy dng thnh mt lut m theo kiu cng khai hm m ho do v chnh bn thn ch nhn ca bc in mt cng gp phi hon cnh tng t nh ngi khc. Nh vy c th gii m mt cch hu hiu th ngi gii m phi c mt hiu bit tuyt mt no v kho gii (mt hiu bit theo kiu nu bit n th cch gii d dng) hiu bit tuyt mt ny c gi l ca sp. Hm mt chiu nh trn c gi l hm mt chiu c ca sp. D nhin d khng bit ca sp th ngi thm m vn c th s dng hiu bit v hm f ln lt tnh tt c cc gi tr f(x) cho mi bn r x cho ti khi tm c bn r tho mn y=f(x). Bn r tm c trn chnh l kt qu gii m ca y. Ngoi ra ngi thm m cn c th s dng nhiu phng php tn cng khc nhm vo c th ring ca tng hm f tm ra bn r trong cc trng hp ring r khc ch khng nht thit phi gii bi ton ngc. Tm li c an ton ca h mt kho cng khai khng ch ph thuc vo kh ca vic gii bi ton ngc m tnh bn ca s an ton ny cn ph thuc vo cc phng php tn cng ca cc thm m, v li nh trnh by trn th ton b cc h kho mt cng khai ang c s dng u cha c s khng nh v tnh kh m ngay c khi c s m bo ny th c s tin b khng ngng ca cng ngh tnh ton tgh hin nhin nhiu vn cha th chp nhn c trong hin ti s c chp nhn trong tng lai. Thc t khng ch i vi cc h mt kho cng khai do vy quan nim mi v tnh an ton tng i m vi n ny sinh ra cc h mt kho cng khai ng thi cng t cho chng ta nhiu bi ton nghim tc phi gii quyt khi s dng h mt ny. Chng ny gii thiu c th mt s h mt cng khai http://www.ebook.edu.vn 74

m vi n s an ton cng nh kh nng ng dng ca n c cc b c v trn th gii tha nhn l h mt kho cng khai sng gi nht, l h mt kho cng khai RSA. Hm m cng khai ek ca Bob phi l mt hm d tnh ton. Song vic tnh hm ngc (tc l hm gii m) phi rt kh khn (i vi bt k ai khng phi l Bob). c tnh d tnh ton nhng kh tnh ngc thng c gi l c tnh mt chiu. Bi vy iu cn thit l ek phi l mt hm mt chiu. Cc hm mt chiu ng mt vai tr trng yu trong mt m hc: Chng rt quan trng trong vic xy dng cc h mt kho cng khai v trong nhiu lnh vc khc. ng tic l, mc d c rt nhiu hm c coi l hm mt chiu nhng cho ti nay vn khng tn ti c mt hm no c th chng minh c l mt hm mt chiu. Sau y l mt v d v mt hm c coi l hm mt chiu. Gi s n l tch ca hai s nguyn p v q, gi s b l mt s nguyn dng. Khi ta xc nh nh x f:Zn Zn l f(x)=xb mod n. (vi b v n c chn thch hp th y chnh l hm m RSA). xy dng mt h mt kho cng khai th vic tm mt hm mt chiu vn cha . Ta khng mun ek l mt hm mt chiu i vi Bob v anh ta phi c kh nng gii m cc bn tin nhn c c hiu qu. iu cn thit l Bob phi c mt ca sp cha thng tin b mt cho php d dng tm ngc ca ek. Nh vy Bob c th gii m mt cch hu hiu v anh ta c mt hiu bit tuyt mt no v K. Bi vy mt hm c gi l ca sp mt chiu nu n l hm mt chiu v n s tr nn d tnh ngc nu bit mt ca sp nht nh. 4.1.2. Nhc li mt s kin thc s hc lin quan nh ngha: Hm Phi Euler ca s nguyn dng n l s cc s nguyn t cng nhau vi n nh hn n.K hiu (n) http://www.ebook.edu.vn 75

V d: (6)=2, (26)=12 Tnh cht ca hm Phi euler: 1. 2. Nu n l s nguyn t th (n) = n-1 Nu p, q l 2 s nguyn t cng nhau thi: V d: (7)=6 (p*q)=(p)*(q) v d (26)=(2*13)=(2)*(13)=1*12=12 3. Nu p l s nguyn t thi: (pr)=(p-1)*pr-1 nh l: Nu a, n l nguyn t cng nhau thi a(n)=1 mod n 4.2. H mt RSA 4.2.1. Thut ton RSA RSA l tn vit tt ca ba tc gi Rivest, Sharmir, Adleman ca trng MIT ra h mt m cng khai. H mt ny c xut nm 1977, da trn c s tnh cc lu tha trong s hc. an ton ca h mt da trn kh ca vic phn tch thnh tha s nguyn t ca cc s nguyn ln. Nhiu h mt kho cng khai sau ny c pht trin nhng u thua km h RSA. Cc h balo ca sp b ph v v cho n nay, ngoi h RSA, cha c mt h no khc cung cp c c an ton v ch k s. a. Thut ton to kho Bc 1: B (ngi nhn) to hai s nguyn t ln ngu nhin p v q (p<>q) Bc 2: B tnh n=p*q v (n) = (p-1)(q-1) Bc 3: B chn mt s ngu nhin e (0 < e < (n)) sao cho CLN(b, (n))=1 Bc 4: B tnh d=e-1 bng cch dng thut ton Euclide Bc 5: B cng b n v e trong danh b lm kho cng khai (public key), cn d lm kho b mt (private key). b. Thut ton m ho v gii m + M ho: http://www.ebook.edu.vn 76

Bc 1: A nhn kho cng khai ca B. Bc 2: A biu din thng tin cn gi thnh s m (0 <= m <= n-1) Bc 3: Tnh c= me mod n Bc 4: Gi c cho B. + Gii m: B gii m bng cch tnh m=cd mod n * Chng minh h mt RSA + Cn chng minh: m = (me mod n)d mod n Tht vy p, q l s nguyn t, n=pq, (n) = (p-1)(q-1) nn ta c m (n) = 1 mod n Mt khc, do ed = 1 mod n nn ed = k (n) + 1 Theo nh l Fermat ta c xp-1 = 1 mod p xq-1 = 1 mod q x(p-1)(q-1) = 1 mod p x(p-1)(q-1) = 1 mod q

x (n) = 1 mod n (me mod n)d mod n = med mod n = mk. (n)+1 mod n = m1 mod n = m (dpcm) * V d: B chn p=5, q=7. Khi n=35, =24 Chn e = 5 (e v nguyn t cng nhau). Letter Encrypt c Decrypt 123.3 4.2.2. Mt s thut ton trin khai trong RSA *Thut ton bnh phng v nhn nh sau: http://www.ebook.edu.vn 77 I 17 I cd m 12 me 1524832 m=cd mod n c=me mod n 17 letter

481968572106750915091411825223072000

Tnh xb mod n Trc ht biu din b= i =0 bi 2i 2 trong bi = 0 hoc 1, 0 i l-1.


l 1

i) z=1 ii) cho i chy t gi tr l-1 v 0 z=z2 mod n Nu bi = 1 th z=z*x mod n iii) gi tr cn tm chnh l gi tr z cui cng. Nh vy s dng thut ton bnh phng v nhn s lm gim s php nhn modulo cn thit, tnh x mod n nhiu nht l 2, trong l l s bt trong biu din nh phn ca b. V l k nn c th coi xb mod n c thc hin trong thi gian a thc 0(k3). * Thut ton clt m rng. Begin g0:= (n) ; g1:=e; u0:=1; u1:=0; v0:=0; v1:=1; While gi 0 do Begin y:=gi-1 div gi ; gi+1:= gi-1 y.gi ; ui+1:= ui-1 y.ui ; vi+1:= vi-1 y.vi ; i:= i+1 ; End; x:= vi-1; If x>0 then d:=x else d:=x+ (n) ; END. V vy mun xy dng h RSA an ton th n=pq phi l mt s ln, khng c kh nng phn tch n v mt tnh ton. m bo an ton nn chn cc s nguyn t p v q t 100 ch s tr ln. http://www.ebook.edu.vn 78

Tuy nhin my tnh thng thng kh c th tnh ton vi s nguyn ln n mc nh vy. Do cn phi c th vin cc thut ton lm vic vi cc s nguyn ln. Ta c th lu tr s ln nh sau: Phn tch s ln thnh s nh phn. Chia s nh phn thnh cc khi 32 bt, lu vo mng, mi phn t

ca mng lu 32 bt. V d: gi s a l s ln c phn tch thnh s nh phn a = a0a1an 32 bt a0 32 bt a1 32 bt an

* Cng hai s ln: S a S b S c a0 b0 c0 a1 b1 c1 an bn cn cn+1

C mt nh 32 bt ghi s nh khi cng 2 s, ban u nh ny bng 0. Khi cng th cc phn t tng ng cng vi nhau nh + a0 + b0 = c0 nh + a1 + b1 = c1 nh + ai + bi = ci xem kt qu c nh hay khng khi tng ci < ai th nh = 1 Mng lu tr tng bao gi cng ln hn mng ca cc s hng tng mt phn t, phn t mng cui cng ny (cn+1) lu s nh. * Nhn s ln Khi nhn 2 s 32 bit s to ra s 64 bt nhng hin nay my tnh khng lu c s 64 bt, nn n chia s 64 bt thnh 2 s 32 bt (32 bt thp v 32 bt cao). Ban u nh = 0.

http://www.ebook.edu.vn

79

32 bt low high

32

bt

Nh vy khi nhn a0 x b0 + nh = c0 (c0 l s 64 bt), s c0 s chia thnh 2 s 32 bt v ghi vo mng c phn t c0 l s 32 bt thp v s nh l 32 bt cao. Phn t tip theo c1 = a0 x b1 + a1 x b0 + nh. c1 cng chia lm 2 s 32 bt v ghi li vo mng c phn t c1 s 32 bt thp v s nh l 32 bt cao. Tng t nh vy ta c tng qut sau:
i

ci = nho + ak bi k
k =0

iu ct yu trong vic thit lp h RSA l to ra cc s nguyn t ln (khong 100 ch s). Qu trnh thc hin trong thc t l : trc ht to ra cc s ngu nhin ln, sau kim tra tnh nguyn t ca n bng cch dng thut ton xc sut Monte Carlo thi gian a thc (nh thut ton Miller Rabin hoc thut ton Solovay Strasen). y l cc thut ton kim tra tnh nguyn t nhanh ca s n trong thi gian a thc theo log2n, l s cc bt trong biu din nh phn ca n). Tuy nhin vn c kh nng thut ton kim tra n l s nguyn t nhng thc t n vn l hp s. Bi vy, bng cch thay i thut ton nhiu ln , c th gim xc sut sai s di mt ngng cho php. Thut ton kim tra s nguyn t: thut ton Miller Rabin Phn tch n 1 = 2k . m , vi m l Chn ngu nhin mt s a sao cho 1 a n-1 Tnh b am mod n. Nu b = 1 th n l s nguyn t v thot. For i:=1 to k-1 do Nu b = -1 th n l s nguyn t, nu khng b = b2 mod n. Tr li n l hp s. 80

http://www.ebook.edu.vn

Xc sut sai lm ca thut ton ny l < 1/4. Trong thc t th cha c bit c mt thust ton kim tra chc chn s sinh ra c phi nguyn t hay khng. Mt vn quan trng khc: l cn phi kim tra bao nhiu s nguyn t ngu nhin (vi kch thc xc nh) cho ti khi tm c mt s nguyn t. Mt kt qu ni ting trong l thuyt s (gi l nh l s nguyn t) pht biu rng: s cc s nguyn t khng ln hn N xp x bng N/lnN. Bi vy, nu p c chn ngu nhin th xc sut p l mt s nguyn t s vo khong 1/lnp. 4.2.3. an ton ca h mt RSA. a. Bi ton phn tch s v vic ph h mt RSA. Cch tn cng d thy nht i vi h mt RSA l ngi thm m s cng gng phn tch n ratha s nguyn t n=p*q v khi anh ta d dng tnh c (n)=(p-1)(q-1) v do tm c thng tin ca sp d tng ng vi thng tin m ho E bng thut ton Euclide. Nh vy chng ta thy ngay rng vic ph h mt RSA l d hn bi ton phn tch s nguyn ra tha s nguyn t tuy nhin cng cha c mt kt qu no ch ra rng bi ton phn tch s l thc s kh hn cho nn ngi ta thn tha nhn rng bi ton ph h RSA l tng ng vi bi ton phn tch s nguyn thnh tha s ngi. m bo tnh kh phn tch ra tha s ca n=p*q th yu cu u tin l p,q l cc s nguyn t ln xp x bng nhau v l s nguyn t mnh . Khi nim mnh y ch bt ngun t ngha kh phn tch do vy n s c b xung cng vi kt qu c c ca kh nng phn tch s. Ni mt cch khc l khi nim mnh bao gm s loi tr cc lp s nguyn t m vi chng tn ti thut ton phn tch hiu qu, chng ta c th bit n mt khi nim s khai ca tnh mnh l cc s nguyn t p m p-1 v p+1 c cha tha s nguyn t ln. b. Vic tn cng h mt RSA khc phng php phn tch s.

http://www.ebook.edu.vn

81

Mt kt qu th v l mt thut ton bt k tnh s m gii m d u c th c dng nh mt chng trnh con trong thut ton xc sut kiu Las Vegas phn tch n. Nh vy mc d rng nu d b l th vic phn tch n cng khng cn ngha theo quan im ph h mt tuy nhin kt qu trn d sao cng cho ta mt thut ton phn tch s n khi bit d vi xc sut thnh cng khng qu ca mi ln chn s ngu nhin lm u vo cho thut ton. 4.2.4. Cc thut ton phn tch s. Trong phn ny gii thiu mt s thut ton phn tch s nguyn c coi l mnh nht theo ngha thi gian tnh tt nht hin nay. Vic trnh by ca chng ti da trn quan im khng phi l a ra thut ton chi tit nhm mc ch phn tch s nguyn m ch yu nu ra tng ca thut ton v quan trng nht l a ra thng s v thi gian tnh ca chng nhm chng minh cho kch thc ti thiu ca cc modulo c s dng trong mt m theo dng tch hai s nguyn t ln. Cc thut ton c k n bao gm thut ton sng bc hai, thut ton phn tch trn ng cong Elliptic, thut ton sng trng s.... nhng do hai thut ton sau u cn phi c kin thc b tr kh cng knh v i s hin i v li iu kin v ti liu li khng chi tit nn bi ging ny ch trnh by thut ton sng bc hai v cng dng nhng nt chnh yu nht. Cc thut ton phn tch s: * Thut ton sng Eratosthenes y l thut ton c tnh ph thng, vi n c c nh th vic p dng thut ton ny l hiu qu. Thi gian tnh ca n l 0( n ). Thut ton c m t nh sau: i) p=1 ii) p=p+1 iii) Tnh r = n mod p. Nu r > 0 quay v bc 2. Ngc li p l c ca N, dng chng trnh. * Thut ton sng ng d http://www.ebook.edu.vn 82

Thut ton c m t nh sau: i) Ly ngu nhin hai s a v b, vi a,b Zn* ii) Kim tra gcd((a-b) mod n,n) >1 hoc gcd((a+b) mod n,n)>1 - Nu ng th gcd((a-b) mod n,n) >1 hoc gcd((a+b) mod n,n)>1 l c ca n dng chng trnh. - Ngc li quay v i) Phn tch thut ton ny di gc xc sut: Cho p l c nguyn t nh nht ca n, th th cn c ti thiu bao nhiu cp a,b c xt n xc sut c t nht mt cp trong s tho mn ((a b) mod p) 0 0.5 ? Bi ton trn c gi l bi ton trng ngy sinh v s m ti thiu cn tm trong bi ton s l m c.p, vi c l mt hng s tnh c no . Thut ton c th thnh cng vi xc sut >0.5, sau khng qu m bc. Bng cch duyt dn th thi gian ca thut ton khng khc g thi gian ca php sng. Tc gi J.M.Pollard s dng mt phng php cn gi l phng php . Ch cn thng qua m bc c th duyt c m cp khc nhau nh nu trn trong thut ton. * Thut ton Pollard Thut ton hiu qu trong vic tm cc c nh l thut ton da vo phng php v c gi l thut ton Pollard. Thi gian tnh ca thut ton ny ch cn l 0( n ). . Vi p l c nguyn t nh nht ca n. Trong trng hp ti nht (p n ) th thi gian tnh ca thut ton cng ch l 4 n Phng php ca Pollard: Tm hai phn t ng d modilo p (a b mod p) nhng khng ng d modulo n. Lc ny p s l c ca gcd(n, (a mb) mod n). C th m t thut ton nh sau: Chn dy gi ngu nhin {xi mod n, i=1,2,} c xc nh nh sau: xi+1 (xi2+ a) mod n vi a 0 v a -2 cn gi tr u x0 tu . Thut ton: i) i=0 ii) i:=i+1 http://www.ebook.edu.vn 83

iii) Xt gcd((x2i xi) mod n,n) > 1 - Nu ng ta c p = gcd((x2i xi) mod n,n). Dng chng trnh - Ngc quay v bc ii) Chng ta i phn tch thi gian ca thut ton: x2i xi (x2i-12 + a) (x2i-1 + a) x22i-1 x2i-1)
(x2i-1 xi-1)(x2i-1+ xi-1)

(x2i-1 + xi-1)(x2i-2 + xi-2)(xi + x0)(xi x0)

Ti bc th i chng ta xt n i+1 cp khc nhau v cng d dng nhn ra rng cc cp c xt trong mi bc l khng ging nhau, do hin nhin vi
p bc chng ra c p cp khc nhau c xt n v nh

phn tch trn. Thut ton thnh cng vi xc sut > 0.5 hay thut ton ca Pollard c thc hin trong 0( n ) bc. * Thut ton p-1 Thut ton p 1 ca Pollard l thut ton phn tch s nguyn n da vo phn tch ca p 1vi p l mt c nguyn t ca n. y l mt thut ton c tc dng nu ta bit c cc c nguyn t ca mt tha s p ca n ni chung v c bit nu n c mt tha s nguyn t p m p 1 ch gm nhng c nguyn t nh nht th thut ton c hiu qu. Thut ton ny ch c hai u vo l n s nguyn l cn c phn tch v mt s b. Cc bc ca thut ton i) u vo l hai s n v b ii) a:=2 iii) for j:=2 to b do a: = aj mod n iv) d = gcd(a-1,n) v) if 1 < d < n then d l mt tha s ca n else khng tm c tha s ca n. V d: Gi s n = 15770708441 v b=180. p dng thut ton p 1 ta c: + a = 1160221425 + d = 135979 http://www.ebook.edu.vn 84

Thc t phn tch y n thnh cc c nguyn t l: N = 15770708441 =135979 x 115979 Php phn tch s thnh cng do 135978 ch gm cc tha s nguyn t nh: 135978 = 2 x 3 x 131 x 173 Trong thut ton c (b-1) lu tha theo modulo, mi lu tha cn nhiu nht l 2log2b php nhn modulo dng thut ton bnh phng v nhn. Vic tm c chung ln nht c th c thc hin trong thi gian 0((log n)3) bng thut ton clt. Bi vy, phc tp ca thut ton l 0(b log b (log n)2 + (logn)3) Nu b l 0((log n)i vi mt s nguyn i xc nh no th thut ton thc s l thut ton thi gian a thc, tuy nhin vi php chn b nh vy, xc sut thnh cng s rt nh. Mt khc, nu tng kch thc ca b ln tht ln th thut ton s thnh cng nhng n s khng nhanh hn php chia th. im bt li ca thut ton ny l n yu cu n phi c c nguyn t p sao cho p - 1 ch c cc tha s nguyn t b. Ta c th xy dng c h mt RSA vi modulo n = p.q hn ch c vic phn tch theo phng php ny. Trc tin tm mt s nguyn t ln p1 sao cho p = 2p1 + 1 cng l mt s nguyn t v mt s nguyn t ln q1 sao cho q = 2q1 + 1 cng l mt s nguyn t. Khi modulo ca RSA n = p.q s chng c cch phn tch theo phng php p 1. * Thut ton p 1 Thut ton p 1 ca Williams cng da vo kt qu phn tch ca p 1 vi p l mt c nguyn t ca n. tin nghin cu phng php p 1, trc ht im li mt s kt qu ca chnh lin quan n dy Lucas nh ngha 1: (dy Lucas) Cho a, b l hai nghim ca phng trnh x2 px + q = 0 (1) K hiu um =
a m bm v vm = a m + b m a b

(2)

Cc dy {um}, {vm}, m = 0, 1, 2, gi l dy Lucas ca phng trnh (1) Ngc li phng trnh (1) gi l phng trnh c trng ca dy (2) http://www.ebook.edu.vn 85

Tnh cht 1: Nu i l c ca j th ui c ca uj Tnh cht 2: Ta c u0 = 0, u1 = 1, v0 = 2, v1 = p v m > 1 th um v vm c tnh theo cng thc sau:
u m +1 vm +1 p Q u1 v1 u v = 1 0 u v 0 0 m m
m

nh l: {um} l dy Lucas ca phng trnh (1) vi p2 4Q = d2 c


khng c c chnh phng (hay bnh phng t do). Nu p khng l c

ca 4Q th u p 0 mod p y l k hiu Legendre p p Thut ton p 1 i) ii) Q=

2 log 2 n...q

log qk

, i = 1, j = 0

Ly khng c c chnh phng ngu nhin trong Zn*. Tm R,

S nguyn sao cho R2 4S = d2 vi d 0 no . Xt gcd( Q, n) > 1 iii) iv) v) vi) Nu ng ta c c ca n l gcd( Q, n). Dng chng trnh Ngc li tnh b u0 mod n ( phn t th Q trong dy Lucas ca Xt ng thc b = 0 Nu ng chuyn sang (iv) Ngc li chuyn sang (vi) Xt j < log q n Nu ng j = j + 1, Q = Q/q quay v (iii) Ngc li chuyn sang (v) Xt i < k Nu ng th : i = i+1, j = 0 Nu b 1 th Q = Q.qi quay v (iv) Ngc lai quay v (i) Xt gcd(b,n) > 1 Nu ng c c ca n l gcd(b,n). Dng chng trnh 86

phng trnh x2 Rx +S = 0)

http://www.ebook.edu.vn

Ngc li quay v (iv)


Ta thy rng vt ht cc kh nng p + 1 (trong trng hp = -1 p v p -1 (trong trng hp = 1)) l c ca Q. Vic xt ng thc b = 0 p trong mi bc, nu sai nhm m bo cho ta b khng l bi ca n v nu p + 1 hoc p 1 l c ca Q th theo cc kt qu tnh cht v nh l trn cho ta b l bi ca p v nh vy gcd(b,n) l c thc s ca n. Tm li, thut ton trn r rng hiu qu trong c hai trng hp p + 1 hoc p 1 ch gm cc c nguyn t nh, tuy nhin cn c vo cng thc tnh cc gi tr ca dy Lucas, ta thy ngay rng h s nhn ca thut ton ny l ln hn nhiu so vi thut ton ca Pollard trong trng hp cng phn tch c n vi c p ca n c p 1 ch gm cc c nh bi v thay cho vic tnh mt lu tha thng thng th thut ton ca Lucas phi tnh mt lu tha ca mt ma trn T thut ton trn, ta c th kt lun: p phi l mt s ln Cc c phi c kch thc xp x nhau Cc c khng c xp x nhau v gi tr c nguyn t p ca modulo n khng c c p + 1 hoc p 1 phn Khng c s Lucas ui = 0 mod p vi i b i vi cc phng trnh P phi c khong cch lu tha 2 ln.

tch hon ton ra cc tha s nguyn t nh c trng c biu thc nh * Phng php le: Phng php le ch c tc dng i vi mt lp s nguyn c bit c th l ch dng phn tch cho cc s nguyn l tch ca cc s nguyn t cng dng r2 + DS2. Thut ton da trn c s l ng thc ca Legendre (cn gi l ng thc Diophantus) ng thc Diophantus: http://www.ebook.edu.vn 87

(x2 + Ly2)(a2 + Lb2) = (x Lyb)2 + L(xb mya)2 Chng minh: Bin i v phi ng thc trn: (xa Ly2) + L(xb mya)2 = x2a2 2Labxy + L2y2b2 + Lx2b2 m2Labxy + Ly2a2 = a2(x2 + Ly2) + Lb2(Ly2 + x2)) = (a2 + Ly2)(x2 + Ly2) Sau le chng minh c rng: nh l: Nu n c hai biu din khc nhau n = r2 + Ls2 = u2 + Lv2 vi gcd() = 1 th n phn tch c thnh tch ca hai tha s n=p.q cng dng p = x2 + Ly2 v q= a2 + Lb2 Nh vy iu kin nhn bit s nguyn n l tch ca hai c s u c dng r2 + Ls2 l n cng c dng v c hai biu din khc nhau theo dng trn. Th nht, ta thy rng t n = r2 + Ls2 nn tm biu din theo dng nu trn ca n ta c th tin hnh bng cch duyt theo s ci nhn bit n Ls2 l s chnh phng. Vi phng php d tm trn th gi tr s ti a cn xt n l
n v y cng l cn tnh ton ca thut ton le. b

Gi s tm c hai biu din khc nhau ca n l: n = r2 + Ls2 = u2 + Lv2. Khng mt tnh tng qut ta coi r, s, u, v khng m v r > u. Khi gii h phng trnh sau y ta tm c x, y, a, b

xa xa xb xb

+ +

L yb = rv L yb = u ya = s ya = v

Du tr ca phng trnh (2) v 93) c ly khi v tri tng ng m. Mt iu kh khn khi thc hin thut ton phn tch le l vn xc nh tham s L. Nhn chung vic thc hin thut ton l ch p dng cho nhng s n m bn thn n bit mt biu din. Tuy nhin li c th bng cch d tm L chng ta c th thnh cng trong vic phn tch. Nh vy thut ton nay ch dng cho mt lp s c bit nn kh c dng to nn mt tiu chun thch hp cho cc modulo hp s. http://www.ebook.edu.vn 88

* Phng php sng Dyxon v sng bc hai Trong phn ny gii thiu thut ton phn tch hai s nguyn c coi l mnh nht theo ngha thi gian tnh tt nht hin nay. tng ca mt lot kh ln cc thut ton phn tch s nh phng php phn tch cc dng chnh phng Danien Shaks, phng php c bit ca le, phng php khai trin lin phn s ca Morrison v Brillhart, phng php sng bc hai ca Pomerance, Dixon l c tm c x y mod n sao cho x2 y2 mod n, cn k thut tm c th nh th no th chnh l ni dung ring ca tng thut ton Thut ton Dixon c thc hin nh sau: S dng mt tp B cha cc s nguyn t b v gi l c s phn tch Chn mt vi s nguyn x sao cho tt c cc tha s nguyn t ca x2 Ly tch ca mt vi gi tr x sao cho mi nguyn t trong c s c

mod n nm trong c s B, s dng mt s chn ln. Chnh iu ny dn n mt ng d thc dng mong mun x2 y2 mod n m ta hy vng s a ti vic phn tch n v suy ra gcd(x-y,n) l mt c ca n. V d: Gi s chn: n = 15770708441, B = {2, 3, 5, 7, 11, 13} V chn ba gi tr x l : 8340934156, 12044942944, 2773700011 Xt ba ng d thc: 83409341562 3x7 (mod n) 120449429442 2x7x13 (mod n) 27737000112 2x3x13 (mod n) Ly tch ca ba ng d thc trn: (8340934156 x 12044942944 x 2773700011)2 (2 x 3 x 7 x 13)2 mod n Rt gn biu thc bn trong du ngoc trong modulo ta c: 95034357852 5462 (mod n) Suy ra

x = 9503435785 y = 546
http://www.ebook.edu.vn 89

Tnh gcd(x-y,n) = gcd(9503435785 546, 15770708441) = 1157759 Ta nhn thy 115759 l mt tha s ca n Gi s: B = {p1,, pB} l mt c s phn tch C ln hn B mt cht (chng hn C = B + 10) C ng d thc:

x2 p1 1j p2 2 j ...pBBj (mod n) j

Vi 1 j C, mi j, xt vc t:
a j = (1 j mod2,2 j mod2,...,Bj mod2) (Z2 )B

Nu c th tm c mt tp con cc aj sao cho tng theo modulo 2 l vect (0, 0,,0) th tch ca cc xj tng ng s c s dng mi nhn t trong B mt s chn ln. V d: Xt li v d trn n = 15770708441, B = {2, 3. 5, 11, 13 Cho ba vect a1, a2, a3 : A1 = (0, 1, 0, 1, 0, 0) A2 = (1, 0, 0, 1, 0, 1) A3 = (1, 1, 0, 0, 0, 1) Suy ra a1 + a2 + a3 = (0, 0, 0, 0, 0, 0) mod 2 Trong trng hp ny nu C<B, vn tm c ph thuc tuyn tnh. y l l do cho thy ng d thc (thit lp theo tch) s phn tch thnh cng c n. Bi ton tm mt tp con C vc t a1, a2, , ac sao cho tng theo modulo 2 l mt vct ton cha s 0 chnh l bi ton tm s ph thuc tuyn tnh (trn Z2) ca vect ny. Vi C > B, s ph thuc tuyn tnh ny nht nh phi tn ti v ta c th d dng tm c bng phng php loi tr Gaux. L do gii thch ti sao ly C > B + 1 l do khng c g m bo mt ng d thc cho trc bt k s to c phn tch n. Ngi ta ch ra rng khong 50% thi gian thut ton cho ra x y (mod n). Tuy nhin nu C > B + 1 th

http://www.ebook.edu.vn

90

c th nhn c mt vi ng d thc nh vy. Hy vng l t nht mt trong cc ng d thc kt qu s dn n vic phn tch n. Vn cn t ra l phi lm nh th no nhn c cc s nguyn xj m cc gi tr xj2 mod n c th phn tch hon ton trn c s B. Mt s phng php c th thc hin c iu . Bin php sng bc hai do Pomerance a ra dng cc s nguyn dng xj = j + n , j = 1, 2, dng xc nh cc xj phn tch c trn B. Nu B l mt s ln th thch hp hn c l nn phn tch s nguyn xj trn B. Khi B cng ln th cng phi gom nhiu ng d thc hn trc khi c th tm ra mt s quan h ph thuc v iu ny dn n thi gian thc hin c

0(e(1+0(1)

ln n lnln n)

Vi 0(1) l mt hm tin ti 0 khi n tin ti

Thut ton sng trng s l thut ton cng phn tch n bng cch xy dng mt ng d thc x2 y2 mod n, song n li c thc hin bng cch tnh ton trn vnh cc s i s. * Thi gian tnh cc thut ton trn thc t Thut ton ng cong Elliptic hiu qu hn nu cc tha s nguyn t ca n c kch thc khc nhau. Mt s rt ln c phn tch bng thut
2 ton ng cong Elliptic l s Fermat (2 1) ( c Brent thc hin nm
n

1988). Thi gian tnh ca thut ton ny c tnh l

0(e

(1+0(1) 2ln p lnln p )

p l tha s nguyn t nh nht ca n Trong trng hp nu hai c ca n chnh lch nhau nhiu th thut ton ng cong Elliptic t ra hn hn thut ton sng bc hai. Tuy nhin nu hai c ca n xp x nhau th thut ton sng bc hai ni chung tri hn thut ton ng cong Elliptic. http://www.ebook.edu.vn 91

Sng bc hai l mt thut ton thnh cng nht khi phn tch cc modulo RSA vi n = p.q v p, q l cc s nguyn t c cng kch thc. Nm 1983, thut ton sng bc 2 phn tch thnh cng s c 69 ch s, s ny l mt tha s ca 2251 1 (do Davis, Holdredye v Simmons thc hin). n nm 1989 c th phn tch c cc s c ti 106 ch s theo thut ton ny ( do Lenstra v Manasse thc hin), nh phn b cc php tnh cho hng trm trm lm vic tch bit ( ngi ta gi phng php ny l Phn tch tha s bng th tn in t). Cc s RSA d vi d l ch s thp phn ca s RSA (d = 100

500)

c cng b trn Internet nh l s thch cho cc thut ton phn tch s. Vo 4/1994 Atkins, Lenstra v Leyland phn tch c mt s 129 ch s, nh s dng sng bc hai. Vic phn tch s RSA 129 trong vng mt nm tnh ton vi my tnh c tc 5 t lnh trn 1 giy, vi cng sc ca hn 600 nh nghin cu trn th gii. Thut ton sng trng s l mt thut ton mi nht trong ba thut ton. Thut ton sng trng s cng phn tch s nguyn n bng vic xy dng ng d thc x2 y2 mod n. Nhng vic thc hin bng cch tnh ton trn cc vnh i s Sng trng s vn cn trong thi k nghin cu. Tuy nhin theo d on th phi chng t nhanh hn vi cc s c trn 125 ch s thp phn. Thi gian tnh ca thut ton sng trng s l

0( e

(1.92 0 (1)) 3 ln n 3 (ln ln n ) 2

Vic trnh by cc thut ton phn tch trn hiu r mt phn no cc bin php tn cng vo RSA c th xy dng mt h mt an ton hn. T cc thut ton trn yu cu i vi p v q nn tho mn: - Cc s nguyn p v q phi xp x nhau v di nhng khng c xp x nhau v ln. - Cc s p 1 v q 1 phi c t nht mt tha s nguyn t ln - Phi c khong lu tha 2 ln - Gi tr F = gcd(p 1, q 1) khng c ln hn 3 n http://www.ebook.edu.vn 92

- Cc s p v q phi l cc s c t nht 100 ch s thp phn Nhn xt u ngn chn kh nng tn cng bi thut ton s ng nht, l thut ton sng, ng thi nh cc phn tch trn th a bi ton phn tch v trng hp kh gii nht, ca ngay thut ton c nh gi l c trin vng nht l thut ton da vo phng php trng s. Nhn xt th hai da vo kh nng ca thut ton Pollard v thut ton Williams m kh nng ph thuc ch yu vo vic cc s p 1 v q 1 phn tch c hon ton qua cc s nguyn t trong tp B. Trong tp B c th l tp cc s nguyn t nh hn 32 bits. Ngc li cng c th s dng tp B ln hn. Do nhn xt ny cng hp l. Vic c mt tham s cng khai nh s m lp m e chc chn phi cung cp thm thng tin cho bi ton phn tch s. Do cn tm hiu mc nh hng ca thng tin ny xy dng nn mt yu cu vi s m e ny v phn no c tnh i ngu lin quan c s m gii m d. cho mt s nguyn t p ng tiu chun v di th i vi h mt s dng bi ton logarit cn cc s nguyn t c di khong gp ri so vi cc s nguyn t dng cho loi h mt da trn bi ton phn tch s. Nu c c mt thut ton nhanh (thut ton xc sut nh Rabin Miller) th thi gian tnh cng phi c 0(n3) ( vi n l di khong gp ri so vi cc s nguyn t trong cc s nh hn n theo Direcle l ( n )
ln n , do vy kh n

nng tm c s nguyn t 521 bt so vi mt s nguyn t 350 bit lu hn gp nhiu ln. Thit k mt h mt s dng bi ton logarit ri rc ch cn ng mt s nguyn t trong khi c mt tnh nng tng ng, th h mt da trn bi ton phn tch s nguyn ra tha s nguyn t cn n 2k s nguyn t cho h thng c k ngi s dng. Cc s nguyn t cn dng cho h mt th hai i hi phi c cc c nguyn t ln, dn n kh nng tm kim s nguyn t cng s kh khn hn nhiu so vi h mt th nht. 4.3. Mt s h mt m cng khai khc http://www.ebook.edu.vn 93

Trong chng ny ta s xem xt mt s h mt kho cng khai khc. H mt Elgamal da trn bi ton logarithm ri rc l bi ton c dng nhiu trong nhiu th tc mt m. Bi vy ta s dnh nhiu thi gian tho lun v bi ton quan trng ny. cc phn sau s xem xt s lc mt s h mt kho cng khai quan trng khc bao gm cc h thong loi Elgamal da trn cc trng hu hn v cc ng cong elliptic, h mt xp ba l MerkleHelman v h mt McElice. 4.3.1.H mt Elgamal v cc logarithm ri rc. H mt Elgamal c xy dng trn bi ton logithm ri rc . Chng ta s bt u bng vic m t bi ton bi khi thit lp mi trng hu hn Zp, p l s nguyn t (Nh li rng nhm nhn Zp* l nhm cyclic v phn t sinh ca Zp* c gi l phn t nguyn thu). Bi ton logarithm ri rc trong Zp l i tng trong nhiu cng trnh nghin cu v c xem l bi ton kh nu p c chn cn thn. C th khng c mt thut ton thi gian a thc no cho bi ton logarithm ri rc. gy kh khn cho cc phng php tn cng bit p phi c t nht 150 ch s v (p-1) phi c t nht mt tha s nguyn t ln. Li th ca bi ton logarithm ri rc trong xy dng h mt l kh tm c cc logarithm ri rc, song bi ton ngc ly lu tha li c th tnh ton hiu qu theo thut ton bnh phng v nhn. Ni cch khc, lu tha theo modulo p l hm mt chiu vi cc s nguyn t p thch hp. Elgamal pht trin mt h mt kho cng khai da trn bi ton logarithm ri rc. H thng ny c trnh by sau. H mt ny l mt h khng tt nh v bn m ph thuc vo c bn r x ln gi tr ngu nhin k do Alice chn. Bi vy, s c nhiu bn m c m t cng bn r. Bi ton logarithm ri rc trong Zp

http://www.ebook.edu.vn

94

c trng ca bi ton: I = (p,,) trong p l s nguyn t, Zp l phn t nguyn thu , Zp* Mc tiu:Hy tm mt s nguyn duy nht a, 0 a p-2 sao cho: a (mod p) Ta s xc nh s nguyn a bng log

H mt kho cng khai Elgamal trong Zp* Cho p l s nguyn t sao cho bi ton logarithm ri rc trong Zp l * * kh gii. Cho Zp l phn t nguyn thu.Gi s P = Zp , * * C = Zp Zp . Ta nh ngha: a K = {(p, ,a,): (mod p)} Cc gi tr p, , c cng khai, cn a gi kn Vi K = (p, ,a,) v mt s ngu nhin b mt k Zp-1, ta xc nh: ek (x,k) = (y1 ,y2 ) trong k y1 = mod p k y2 = x mod p * vi y1 ,y2 Zp ta xc nh: a -1 dk(y1 ,y2 ) = y2 (y1 ) mod p Sau y s nm t s lc cch lm vic ca h mt Elgamal .Bn r x c che du bng cch nhn n vi k to y2 . Gi tr k cng c gi i nh mt phn ca bn m. Bob ngi bit s m b mt a c th tnh c k t k . Sau anh ta s tho mt n bng cch chia y2 cho k thu c x. V d: Cho p = 2579, = 2, a = 765. Khi = 2765 mod 2579 = 949 By gi ta gi s Alice mun gi thng bo x = 1299 ti Bob. Gi s s ngu nhin k m c chn l k = 853. Sau c ta tnh http://www.ebook.edu.vn 95

y1 = 2853 mod 2579 = 435 y2 = 1299 949853 mod 2579 = 2396 Khi Bob thu c bn m y = (435,2396), anh ta tnh x = 2396 (435765)-1 mod 2579 = 1299 chnh l bn r m Alice m ho. 4.3.2 Mt m Bal. 4.3.2.1. C s ca mt m bal Mt m bal xut pht t bi ton tng tp con tng qut (bi ton Bi ton c pht biu nh sau: Cho dy cc s dng S={s1, s2,., sn} v mt s dng C. Hi c tn ti mt tp con nm trong S sao cho tng tp con bng C. (Hi c tn ti mt vc t nh phn x=(x1, x2,, xn) sao cho C=xi.si (i=1..n)) y l bi ton kh c thi gian l hm m O(2n). Nu S l dy siu tng th bi ton trn gii c vi thi gian tuyn tnh O(n). nh ngha: Dy S gi l siu tng nu mi si>sj (j=1,..i-1) (tc l phn t ng sau ln hn tng cc phn t ng trc n) Khi bi ton tng tp con c pht biu nh sau: Cho dy siu tng S={s1, s2,., sn} v mt s dng C. Hi c tn ti mt tp con nm trong S sao cho tng tp con bng C. (Hi c tn ti mt vc t nh phn x=(x1, x2,, xn) sao cho C=xi.si (i=1..n)) Khi bi ton c gii nh sau: For i:=n downto 1 do Begin If C>=si then xi=1 http://www.ebook.edu.vn 96 al ).

Else xi:=0; C:=C-xi.si; End; If C=0 then bi ton c p n l vc t x Else bi ton khng c p n; p dng bi ton ny ta s dng dy S siu tng lm kha b mt. Sau tc ng ln dy S bin i thnh mt dy bt k, v cng khai dy ny l kha cng khai. Ta c h mt m 4.3.2.2. Thut ton: * To kha: - Chn dy siu tng S={s1, s2, , s3} - Chn p sao cho p>si (i=1..n) - Chn a sao cho 1<a<p-1 v (a,p)=1; - tnh t=a.s mod p => kha cng khai l t, kha b mt l: a, p, S * M: Chn bn r l dy nh phn x=(x1, x2,, xn) Tnh bn m y=xi.ti (i=1..n) Gi bn m y * Gii m: - Tnh C=a-1.y mod p - Gii bi ton ba l vi S l dy siu tng v s dng C tm bn r x * Chng minh tnh ng ca h mt m ba l (Bn c t chng minh) V d: (Nh mt bi tp). al nh sau:

http://www.ebook.edu.vn

97

Chng 5 Cc s ch k s 5.1. Gii thiu. Trong chng ny, chng ta xem xt cc s ch k s (cn c gi l ch k s). Ch k vit tay thng thng trn ti liu thng c dng xc ngi k n. Ch k c dng hng ngy chng hn nh trn mt bc th nhn tin t nh bng, k hp ng S ch k l phng php k mt bc in lu di dng in t. Chng hn mt bc in c k hiu c truyn trn mng my tinh. Chng ny trnh by mt vi s ch k s. Ta s tho lun trn mt vi khc bit c bn gia cc ch k thng thng v ch k s. u tin l mt vn k mt ti liu. Vi ch k thng thng, n l mt phn vt l ca ti liu. Tuy nhin, mt ch k s khng gn theo kiu vt l vo bc in nn thut ton c dng phi khng nhn thy theo cch no trn bc in. Th hai l vn v kim tra. Ch k thng thng c kim tra bng cch so snh n vi cc ch k xc thc khc. V d, ai k mt tm sc mua hng, ngi bn phi so snh ch k trn mnh giy vi ch k nm mt sau ca th tn dng kim tra. D nhin, y khng phi l phg php an ton v n d dng gi mo. Mt khc, cc ch k s c th c kim tra nh dng mt thut ton kim tra cng khai. Nh vy, bt k ai cng c th kim tra dc ch k s. Vic dng mt s ch k an ton c th s ngn chn c kh nng gi mo. S khc bit c bn khc gia ch k s v ch k thng thng bn copy ti liu c k bng ch k s ng nht vi bn gc, cn copy ti liu c ch k trn giy thng c th khc vi bn gc. iu ny c ngha l phi cn thn ngn chn mt bc k s khi b dung li. V th, bn thn bc in cn cha thng tin (chng hn nh ngy thng) ngn n khi b dng li. Mt s ch k s thng cha hai thnh phn: thut ton k v thut ton xc minh. Bob c th k bc in x dng thut ton k an ton. Ch k http://www.ebook.edu.vn 98

y=sig(x) nhn c c th kim tra bng thut ton xc minh cng khai ver(x,y). Khi cho trc cp (x,y), thut ton xc minh c gi tr TRUE hay FALSE tu thuc vo ch k c thc nh th no. Di y l nh ngha hnh thc ca ch k: nh ngha: Mt s ch k s l b 5( P, A, K, S, V) tho mn cc iu kin di y: 1. 2. 3. P l tp hu hn cc bc in c th. A l tp hu hn cc ch k c th. K khng gian kho l tp hu hn cc kho c th. 4. Vi mi k thuc K tn ti mt thut ton k sigk S v l mt thut ton xc minh verk V. Mi sigk : P A v verk: Pa {true,false} l nhng hm sao cho mi bc in x P v mi ch k y A tho mn phng trnh di y. True nu y=sig(x) verk False nu y# sig(x) Vi mi k thuc K hm sigk v verk l cc hm thi than a thc. Verk

s l hm cng khai sigk l mt. Khng th d dng tnh ton gi mo ch k ca Bob trn bc in x. Ngha l x cho trc, ch c Bob mi c th tnh c y verk = True. Mt s ch k khng th an ton v iu kin v Oscar c th kim tra tt c cc ch s y c th c trn bc in x nh ung thut ton ver cng khai cho n khi anh ta tm thy mt ch k ng. Vi th, nu c thi gian. Oscar lun lun c th gi mo ch k ca Bob. Nh vy, ging nh trng hp h thng m kho cng khai, mc ch ca chng ta l tm cc s ch k s an toan v mt tnh ton. Xem thy rng, h thng m kho cng khai RSA c th ch k s. ung lm s

http://www.ebook.edu.vn

99

Nh vy, Bob k bc in x dng qui tc gii m RSA l dk. Bob l ngi to ra ch k v dk = sigk l mt. Thut ton xc minh dng qui tc m RSA ek. Bt k ai cng c th xc minh ch k vi ek c cng khai. Ch rng, ai c th gi mo ch k ca Bob trn mt bc in ngu nhin x bng cch tm x=ek(y) vi y no , khi y= sigk(x). Mt gii php xung quanh vn kh khn ny l yu cu bc in cha phn d ch k gi mo kiu ny khng tng ng vi bc in. Ngha l x tr mt xc sut rt b. C th dng cc hm hash trong vic kt ni vi cc s ch k s s loi tr c phng php gi mo ny. S ch k RSA Cho n= p.q, p v q l cc s nguyn t. Cho P =A= Zn ab 1(mod( (n))). Cc gi tr n v b l cng khai, a gi b mt. Hm k: sigk(x)= xa mod n v kim tra ch k: verk (x,y)= true x yb (mod n) (x,y Zn) Ta xt tm tt cch kt hp ch k v m kho cng khai. Gi s rng, Alice tnh ton ch k y= sigAlice(x) v sau m c x v y bng hm m kho cng khai eBob ca Bob, khi c ta nhn c z = eBob(x,y). Bn m z s c truyn ti Bob. Khi Bob nhn c z, anh ta s trc ht s gii m hm dBob nhn c (x,y). Sau anh ta ung hm xc minh cng khai ca Alice kim tra xem verAlice(x,y) c bng True hay khng. Song nu u tin Alice m x ri sau mi k tn bn m nhn c th khi c tnh : y= sigAlice(eBob(x)). Alice s truyn cp (z,y) ti Bob. Bob s gii m z, nhn x v sau xc minh ch k y trn x nh dng verAlice. Mt vn tim n trong bin php ny l nu Oscar nhn c cp (x,y) kiu ny, c ta c thay ch k y ca Alice bng ch k ca mnh. Y, = sigOscar(eBob(x)). http://www.ebook.edu.vn 100

(Ch , Oscar c th k bn m eBob(x) ngay c khi anh ta khng bit bn r x). Khi nu Oscar truyn (x, y ) n Bob th ch k Oscar c Bob xc minh bng verOscar v Bob c th suy ra rng, bn r x xut pht t Oscar. Do kh khn ny, hu ht ngi s dng c khuyn ngh nu k trc khi m. 5.2. S ch k ELGAMAL Sau y ta s m t s ch k Elgamal tng di thiu trong bi bo nm 1985. Bn c tin ca s ny c Vin Tiu chun v Cng Ngh Quc Gia M (NIST) chp nhn lm ch k s. S Elgamal (E.) c thit k vi mc ch dnh ring cho ch k s, khc s RSA dng cho c h thng m kho cng khai ln ch k s. S E, l khng tt nh ging nh h thng m kho cng khai Elgamal. iu ny c ngha l c nhiu ch k hp l trn bc in cho trc bt k. Thut ton xc minh phi c kh nng chp nhn bt k ch k hp l khi xc thc. Nu ch k c thit lp ng khi xc minh s thnh cng v : a k(mod p) x(mod p) l y ta dng h thc : a + k x (mod p-1) S ch k s Elgamal.

Cho p l s nguyn t sao cho bi ton logarit ri rc trn Zp l kh v * * gi s Zn l phn t nguyn thu p = Zp , a = Zp Zp-1 v nh ngha: a K ={(p, ,a, ): (mod p)}. Gi tr p, , l cng khai, cn a l mt. Vi K = (p, , a, ) v mt s ngu nhin (mt) k Zp-1. nh ngha : Sigk(x,y) =( ,), k trong = mod p -1 v =(x-a) k mod (p-1). Vi x, Zp v Zp-1 , ta nh ngha : Ver(x, , http://www.ebook.edu.vn ) = true 101 (mod p).
x

Bob tnh ch k bng cch dng c ga tr mt a (l mt phn ca kho) ln s ngu nhin mt k (dng k ln bc in x). Vic xc minh c thc hin duy nht bng thng bo tin cng khai. Chng ta hy xt mt v d nh minh ho. Gi s cho p = 467, =2, a = 127, khi : = a mod p = 2127 mod 467 = 132 Nu Bob mun k ln bc in x = 100 v chn s ngu nhin k =213 (ch l UCLN(213,466) =1 v 213-1 mod 466 = 431. Khi =2213 mod 467 = 29 v =(100-127 29) 431 mod 466 = 51. 13229 2951 189 (mod 467) v 2100 189 (mod 467) V th ch k l hp l. Xt mt ca s ch k E. Gi s, Oscar th gi mo ch k trn bc in x cho trc khng bit a. Nu Oscar chn v sau th tm gi tr tng ng, anh ta phi tnh logarithm ri rc log x-. Mt khc, nu u tin ta chn v sau th tim v th gii phng trnh: x(mod p). tm . y l bi ton cha c li gii no. Tuy nhin, dng nh n cha c gn vi n bi ton nghin cu k no nn vn c kh nng c cch no tnh v ng thi (, ) l mt ch k. Hin thi khng ai tm c cch gii song cng ai khng khng nh c rng n khng th gii c. http://www.ebook.edu.vn 102 Bt k ai cng c th xc minh ch k bng cc kim tra :

Nu Oscar chn v v sau t gii tm x, anh ta s phi i mt vi bi ton logarithm ri rc, tc bi ton tnh log V th Oscar khng th k mt bc in ngu nhin bng bin php ny. Tuy nhin, c mt cch Oscar c th k ln bc in ngu nhin bng vic chn , v x ng thi: gi thit i v j l cc s nguyn 0 i p-2, 0 j p-2 v UCLN(j,p-2) = 1. Khi thc hin cc tnh ton sau: = i j mod p = - j-1 mod (p-1) x = - i j-1 mod (p-1) Trong j-1 c tnh theo modulo (p-1) ( y i hi j nguyn t cng nhau vi p-1). Ta ni rng (, ) l ch k hp l ca x. iu ny c chng minh qua vic kim tra xc minh : Ta s minh ho bng mt v d : Ging nh v d trc cho p = 467, = 2, =132. Gi s Oscar chn i = 99,j = 179; khi j-1 mod (p-1) = 151. Anh ta tnh ton nh sau: = 299132197 mod 467 = 117 =-117 151 mod 466 = 51. x = 99 41 mod 466 = 331 Khi (117, 41) l ch k hp l trn bc in 331 nh th xc minh qua php kim tra sau: 132117 11741 303 (mod 467) v 2331 303 (mod 467)

V th ch k l hp l. Sau y l kiu gi mo th hai trong Oscar bt u bng bc in c Bob k trc y. Gi s (, ) l ch k hp l trn x. Khi Oscar c kh nng k ln nhiu bc in khc nhau. Gi s i, j, h l cc s nguyn, 0 h, i, j p-2 v UCLN (h - j , p-1) = 1. Ta thc hin tnh ton sau: http://www.ebook.edu.vn 103

= h i j mod p = (h -j)-1 mod (p-1) x, = (hx+i ) -1 mod (p-1), Trong (h -j)-1 c tnh theo modulo (p-1). Khi d dng kim tra iu kin xc minh : x (mod p) v th (, )l ch k hp l ca x. C hai phng php trn u to cc ch k gi mo hp l song khng xut hin kh nng i phng gi mo ch k trn bc in c s lu chn ca chnh h m khng phi gii bi ton logarithm ri rc, v th khng c g nguy him v an ton ca s ch k Elgamal. Cui cng, ta s nu vi cch c th phi c s ny nu khng p dng n mt cch cn thn (c mt s v d na v khim khuyt ca giao thc, mt s trong l xt trong chng 4). Trc ht, gi tr k ngu nhin c dng tnh ch k phi gi kn khng l. v nu k b l, kh n gin tnh : A = (x-k )-1 mod (p-1). D nhin, mt khi a b l th h thng b ph v Oscar c th d dang gi mo ch k. Mt kiu dung sai s na l dng cng gi tr k k hai bc in khc nhau. iu ny cng to thun li cho Oscar tinh a v ph h thng. Sau y l cch thc hin. Gi s (, 1) l ch k trn x1 v (, 2) l ch k trn x2. Khi ta c: 1 x1 (mod p) v Nh vy x1-x2 1-2 (mod p). Nu vit = k, ta nhn c phng trnh tm k cha bit sau. http://www.ebook.edu.vn 104 2 x2(modp).

x1-x2 k(1 -2) (mod p) tng ng vi phng trnh x1- x2 k( 1- 2) (mod p-1). By gi gi s d =UCLN(1- 2, p-1). V d | (p-1) v d | (1-2) nn suy ra d | (x1-x2). Ta nh ngha: x = (x1- x2)/d = (1- 2)/d p = ( p -1 )/d Khi ngd thc tr thnh: x k (mod p ) v UCLN(, p ) = 1,nn c th tnh: = ()-1 mod p Khi gi tr k xc nh theo modulo p s l: k = x mod p Phng trnh ny cho d gi tr c th ca k k = x +i p mod p vi i no , 0 i d-1. Trong s d gi tr c c th ny, c th xc nh c mt gi tr ng duy nht qua vic kim tra iu kin k (mod p) 5.3. Chun ch k s. Chun ch k s(DSS) l phin bn ci tin ca s ch k Elgamal. N c cng b trong H S trong lin bang vo ngy 19/5/94 v c lm http://www.ebook.edu.vn 105

chun vo 1/12/94 tuy c xut t 8/91. Trc ht ta s nu ra nhng thay i ca n so vi s Elgamal v sau s m t cch thc hin n.Trong nhiu tinh hung, thng bo c th m v gii m ch mt ln nn n ph hp cho vic dng vi h mt bt k (an ton ti thi im c m). Song trn thc t, nhiu khi mt bc in c dng lm mt ti liu i chng, chng hn nh bn hp ng hay mt chc th v v th cn xc minh ch k sau nhiu nm k t lc bc in c k. Bi vy, iu quan trng l c phng n d phng lin quan n s an ton ca s ch k khi i mt vi h thng m. V s Elgamal khng an ton hn bi ton logarithm ri rc nn cn dung modulo p ln. Chc chn p cn t nht l 512 bt v nhiu ngi nht tr l p nn ly p=1024 bt c an ton tt. Tuy nhin, khi ch ly modulo p =512 th ch k s c 1024 bt. i vi nhiu ng dng dng th thng minh th cn li c ch k ngn hn. DSS ci tin s Elgamal theo hng sao cho mt bc in 160 bt c k bng ch k 302 bt song li p = 512 bt. Khi h thng lm vic trong nhm con Zn* kch thc 2160. mt ca h thng da trn s an ton ca vic tm cc logarithm ri rc trong nhm con Zn*. S thay i u tin l thay du - bng + trong nh ngha , v th: = (x + )k-1 mod (p-1) thay i ko theo thay i iu kin xc minh nh sau: x (mod p) (6.1) Nu UCLN (x + , p-1) =1th -1 mod (p-1) tn ti v ta c th thay i iu kin (6.1) nh sau:
-1 -1 x (mod )p (6.2)

y l thay i ch yu trong DSS. Gi s q l s nguyn t 160 bt sao cho q | (q-1) v l cn bc q ca mt modulo p. (D dng xy dng mt nh vy: cho 0 l phn t nguyn thu ca Zp v nh ngha = 0(p-1)/q mod p). http://www.ebook.edu.vn 106

Khi v cng s l cn bc q ca 1. v th cc s m Bt k ca , v c th rt gn theo modulo q m khng nh hng n iu kin xc minh (6.2). iu rc ri y l xut hin di dng s m v tri ca (6.2) song khng nh vy v phi. V th, nu rt gn theo modulo q th cng phi rt gn ton b v tri ca (6.2) theo modulo q thc hin php kim tra. Nhn xt rng, s (6.1) s khng lm vic nu thc hin rt gn theo modulo q trn (6.1). DSS c m t y trong s di. Ch cn c 0 (mod q) v gi tr -1 mod q cn thit xc minh ch k (iu ny tng vi yu cu UCLN(, p-1 ) =1 khi bin i (6.1) thnh (6.2). Nu Bob tnh 0 (mod q) theo thut ton ch k, anh ta s loi i v xy dng ch k mi vi s ngu nhin k mi. Cn ch ra rng, iu ny c th khng gn vn trn thc t: xc xut 0 (mod q) chc s xy ra c 2-160 nn n s hu nh khng bao gi xy ra. Di y l mt v d minh ho nh Chun ch k s. Gi s p l s nguyn t 512 bt sao cho bi ton logarithm ri rc trong Zp khng gii c, cho p l s nguyn t 160 bt l c ca (p-1). Gi thit Zp l cn bc q ca 1modulo p: Cho p =Zp . a = Zq Zp v nh ngha : a A = {(p,q, ,a, ) : (mod p)} cc s p, q, v l cng khai, c a mt. Vi K = (p,q, ,a, )v vi mt s ngu nhin (mt) k ,1 k q-1, ta nh ngha: sigk (x,k) = ( ,) k trong =( mod p) mod q -1 v = (x +a )k mod q Vi x Zp v , Zq , qua trnh xc minh s hon ton sau cc tnh ton : e1= x-1 mod q e2= -1 mod q verk(x, , ) = true ( e1e2 mod p) mod q =

http://www.ebook.edu.vn

107

V d: Gi s q =101, p = 78 q+1 =7879.3 l phn t nguyn thu trong Z7879 nn ta c th ly: = 378 mod 7879 =170 Gi s a =75, khi : = a mod 7879 = 4576 By gi gi s Bob mun k bc in x = 1234 v anh ta chn s ngu nhin k =50, v th : k-1 mod 101 = 99 khi = 2518 mod 101 = 94 v = (1234 +75 94) mod 101 = 96 Ch k (94, 97) trn bc in 1234 c xc minh bng cc tnh ton sau: -1 = 97-1 mod 101 =25 e1 = 1234 25mod 101 = 45 e2 = 94 25 mod 101 =27 (17045 456727 mod 7879)mod =2518 mod 101 = 94 v th ch k hp l. Khi DSS c xut nm 1991, c mt vi ch trch a ra. Mt kin cho rng, vic x l la chn ca NIST l khng cng khai. Tiu chun c Cc An ninh Quc gia (NSA) pht trin m khng c s tham gia ca khi cng nghip M. Bt chp nhng u th ca s , nhiu ngi ng cht ca khng tip nhn. Cn nhng ch trch v mt k thut th ch yu l v kch thc modulo p b c nh = 512 bt. Nhiu ngi mun kch thc ny c th thay i c nu cn, c th dng kch c ln hn. p ng nhng i hi ny, NIST chn tiu chun cho php c nhiu c modulo, ngha l c modulo bt k chia ht cho 64 trong phm vi t 512 n 1024 bt. http://www.ebook.edu.vn 108 =(17030 mod 7879) mod 101

Mt phn nn khc v DSS l ch k c to ra nhanh hn vic xc minh n. Trong khi , nu dng RSA lm s ch k vi s m xc minh cng khai nh hn (chng hn = 3) th c th xc minh nhanh hn nhiu so vi vic lp ch k. iu ny dn n hai vn lin quan n nhng ng dng ca s ch k: 1.Bc in ch c k mt ln, song nhiu khi li cn xc minh ch k nhiu ln trong nhiu nm. iu ny li gi nhu cu c thut ton xc minh nhanh hn. 2.Nhng kiu my tnh no c th dng k v xc minh ? Nhiu ng dng, chng hn cc th thng minh c kh nng x l hn ch li lin lc vi my tnh mnh hn. Vi th c nhu cu nhng thit k mt s c thc hin trn th mt vi tnh ton. Tuy nhin, c nhng tnh hung cn h thng mnh to ch k, trong nhng tnh hung khc li cn th thng minh xc minh ch k. V th c th a ra gii php xc nh y. S p ng ca NIST i vi yu cu v s ln to xc minh ch k thc ra khng c vn g ngoi yu cu v tc , min l c hai th thc hin nhanh.

http://www.ebook.edu.vn

109

You might also like